• Shuffle
    Toggle On
    Toggle Off
  • Alphabetize
    Toggle On
    Toggle Off
  • Front First
    Toggle On
    Toggle Off
  • Both Sides
    Toggle On
    Toggle Off
  • Read
    Toggle On
    Toggle Off
Reading...
Front

Card Range To Study

through

image

Play button

image

Play button

image

Progress

1/112

Click to flip

Use LEFT and RIGHT arrow keys to navigate between flashcards;

Use UP and DOWN arrow keys to flip the card;

H to show hint;

A reads text to speech;

112 Cards in this Set

  • Front
  • Back

Under what circumstances must a pleading include verification under oath that the pleading is true to the knowledge of the deponent?


A. Answer to a charge of common-law fraudB. Matrimonial action for divorce, separation, or annulmentC. Response to a verified pleadingD. All of the above

Correct: Answer choice D is correct. In only a few instances is verification of a pleading mandatory. However, such instances include a complaint in a matrimonial action, an answer to a complaint charging common-law fraud, and a response to any pleading that is verified. Because verified pleadings are required in the situations listed in A, B, and C, those individual answer choices are incorrect.

Defendant ordered cocaine from his supplier, and the supplier's courier delivered the cocaine to Defendant's home. Upon inspection of the cocaine, Defendant rejected it because of perceived defects in the quality. Upon evidence legally sufficient to establish this sequence of events, the prosecution charged Defendant with possession and attempted possession of cocaine. Defendant moved to dismiss the charges. How should the court rule?


A. Uphold the charge of possession because the Defendant's inspection was sufficient to constitute possession.B. Uphold the charge of attempted possession, because Defendant's conduct evidenced sufficient intent and an act in furtherance of that objective.C. Dismiss the charge of attempted possession because it is legally impossible to attempt to possess without actually possessing it.D. Dismiss both charges, because Defendant's conduct is insufficient to evidence either possession or attempted possession of cocaine.

Correct: Answer choice B is correct. A person knowingly and unlawfully possessing a substance weighing at least four ounces and containing a narcotic drug is guilty of criminal possession of a controlled substance in the first degree. N.Y. Penal Law § 220.21(1). Under the penal law, "[a] person is guilty of an attempt to commit a crime when, with intent to commit a crime, he engages in conduct which tends to effect the commission of such crime." N.Y. Penal Law § 110.00. Although a person who orders illegal narcotics from a supplier, admits a courier into his home, and examines the quality of the goods doesn't actually possess those drugs (making answer choice A incorrect), he has unquestionably passed beyond mere preparation and has engaged in conduct tending to complete the commission of the crime (i.e., possessing the drugs). Indeed, the only remaining step between the attempt and the completed crime is the person's acceptance of the proffered merchandise, an act entirely within his control. Thus, answer choices C and D are incorrect.

In January, an initial certificate of limited partnership to form Limited Partnership was created and entrusted to partnership's attorney for filing. Unbeknownst to Limited Partnership, the attorney failed to file the certificate with the department of state. In July, Limited Partnership opened an investment account with Firm. Within one year of opening the account, Limited Partnership's investments lost more than half their value, and Limited Partnership commenced an action against Firm seeking damages for negligence in managing its funds. A summons and complaint were served on Firm in October of the following year. Two years later, Limited Partnership filed the initial certificate upon discovery that their attorney had failed to do so. Firm moved to dismiss the complaint, asserting that the consequence of Limited Partnership's failure to file the initial certificate prior to commencement of the action resulted in a lack of capacity to enter into an agreement or bring suit.
Will Firm prevail in its motion to dismiss?


A. Yes, because filing a certificate of limited partnership is mandatory and conclusive evidence that a limited partnership is in existence.B. Yes, because failure to comply with legal requirements estops a litigant from defending itself in a lawsuit before the state court.C. No, because Firm derived a benefit from its agreement with Limited Partnership, and its investment services were not dependent in any way on the nature of Limited Partnership.D. No, because the only requirement for formation of a Limited Partnership is an oral or written agreement between the parties.

Answer choice C is correct. Filing a certificate is conclusive evidence of the formation of a limited partnership and is required by the Revised Uniform Limited Partnership Act (RULPA); thus, answer choice D is incorrect. However, one who has recognized the organization in business dealings may not be allowed to deny its existence in matters before the court and may be estopped from doing so. Here, Firm derived a benefit from the agreement, and the services it provided did not depend on the certificate of limited partnership being properly filed. Therefore answer choices A and B are incorrect, and Firm is estopped from denying Limited Partnership's validity. Note that if Limited Partnership were the party seeking to file a lawsuit, the court would be more likely to find it was estopped as a result of its failure to properly file its certificate of formation.


Infant was intentionally hit in the head by six-year-old Child. Child swung an aluminum bat that Child’s mother had provided for play in her backyard, and the bat caused Infant to suffer a concussion. Infant’s parents seek to bring a lawsuit on behalf of Infant for her injuries. Which theory of recovery would you advise Infant’s parents is most likely to be successful against Child’s mother?


A. Negligent supervision of a childB. Negligent entrustment of a dangerous instrumentC. BatteryD. None of the above

Answer choice B is correct. New York does not recognize a cause of action against parents for negligent supervision. Thus, A is incorrect. However, a parent may be liable for negligently entrusting a child with a dangerous instrument. Thus, D is incorrect. Should a jury determine that an aluminum bat is a dangerous instrument, negligent entrustment may be a viable theory of recovery. Battery is not a viable cause of action because the intentional and offensive touching here was caused by Child, rather than Child’s mother, who is the named defendant. Parents are not vicariously liable for their minor child’s torts unless the child commits a tort while acting as the parent’s agent.

Landlord and Tenant entered into a lease in which Landlord agreed to lease premises to Tenant for a period of five years. After two years, Tenant received an opportunity to move overseas. Friend was interested in assuming the lease or subletting the premises from Tenant. Landlord and Tenant's lease was silent as to whether the lease could be assigned or the premises sublet. Tenant contacted Landlord by telephone and mail for permission to assign the lease or sublet the premises to Friend. Landlord had no reason to deny the request, but nonetheless ignored Tenant's repeated requests for permission to assign or sublet the premises. Tenant has come to you for advice. How would you advise?
How would you advise?


A. Tenant may neither assign nor sublet the premises without Landlord's written consent.B. Tenant may assign or sublet the premises because, absent an express provision to the contrary, leases are freely assignable or can be freely sublet.C. Tenant may assign but not sublet the premises because leases are freely assignable without the landlord's consent, but sublets require a landlord's written authorization.D. Tenant may sublet but not assign the premises because an assignment of a lease may not occur without a landlord's written consent, but failure to respond to a tenant's request to sublease is deemed to be consent.

Answer choice D is correct. In New York, unless otherwise agreed, no assignment can occur without the landlord's written consent. Landlords may refuse to consent to an assignment of the lease for any reason or cause. Thus, B and C are incorrect. Like with an assignment, a sublease cannot occur without the landlord's consent. However, unlike with an assignment, a landlord must have a reason to deny consent to a sublease. Failure to respond to a tenant's request to sublease is deemed to be consent. NY Real Prop. Law Sec. 226-b.

Plaintiff obtained a judgment against Defendant for $1,000,000 in a defamation claim. Which of the following items of personal property are exempt from being used to satisfy the money judgment?


A. One computerB. One televisionC. A watch not exceeding $1,000D. All of the above

Answer choice D is correct. CPLR 5205 (a) exempts all of the above listed personal property from application to the satisfaction of a money judgment. Accordingly, answer choices A, B, and C are incorrect.


Corporation was a defendant in litigation with Plaintiff. Although Corporation employed inside counsel to draft contracts and ensure corporate compliance, it retained an outside law firm to represent it in the litigation with Plaintiff.

May Plaintiff’s attorney communicate directly with Corporation’s inside counsel concerning matters related to the litigation without violating the no-contact rule?


A. No, the no-contact rule applies to Corporation’s employees, irrespective of whether such employees are attorneys.B. No, because employment of in-house counsel is not sufficient to presume that counsel is skilled and knowledgeable in protecting Corporation’s interests.C. Yes, unless the no-contact rule has been expressly invoked by the law firm representing Corporation in the litigation.D. Yes, provided there is a reasonable, good-faith belief that in-house counsel has authority to serve as lawyer for Corporation in the litigation.

Answer choice D is correct. A lawyer is prohibited from communicating with a party known to be represented by another lawyer without consent. Nevertheless, a lawyer is not prohibited from communicating with an in-house counsel of a party known to be represented in that matter, so long as the lawyer seeking to make that communication has a reasonable, good-faith belief based on objective indicia that such an individual is serving as a lawyer for the entity. If in-house counsel is a represented party (i.e., a client of the outside lawyer), then RPC 4.2 will automatically prohibit contact with in-house counsel by Plaintiff’s attorney. Thus, answer choice C is incorrect. On the other hand, if in-house counsel was representing Corporation, then such contact would not be prohibited. Thus, answer choice A is incorrect. The principal purposes of the Rule are to prevent a lawyer from taking advantage of a non-lawyer who is represented by counsel and to preserve the attorney-client relationship once it has been established. It may be presumed that in-house counsel, trained in the law, can exercise judgment as to whether he should engage in a given communication. Thus, answer choice B is incorrect.

Plaintiff was injured in a rear-end collision while waiting for a red light to turn green. Plaintiff’s injuries were exacerbated by the fact that she had neglected to wear a seat belt. Based on Plaintiff’s failure to wear a seat belt, Defendant alleged two affirmative defenses: comparative negligence and an assumption of the risk. Plaintiff moves to have both of the affirmative defenses dismissed. How should the court rule?


A. Grant the motion to dismiss the affirmative defense of comparative negligence, but allow a trier of fact to determine whether Plaintiff assumed the risk.B. Grant the motion to dismiss the affirmative defense of assumption of risk, but allow a trier of fact to determine whether Plaintiff’s failure to wear a seat belt constitutes comparative negligence.C. Deny the motion to dismiss in its entirety, and allow the trier of fact to determine whether Plaintiff’s failure to wear a seat belt constituted comparative negligence or assumption of the risk.D. Grant the motion to dismiss in its entirety.

Answer choice D is correct. In New York, the plaintiff’s failure to wear a seatbelt is not admissible as evidence of comparative negligence, and thus B and C are incorrect. Further, in order for assumption of risk to be viable, the plaintiff must be voluntarily engaged in an activity with an elevated risk of injury that is obvious and inherent in the activity, or that plaintiff has knowledge of and expressly assumes. This defense is applicable to suits involving sporting injuries or entertainment event, but is not appropriate in an instance involving a driver of a motor vehicle waiting at a traffic light. Thus, A is incorrect. Note, however, that Defendant would not be liable for Plaintiff’s injuries that could have been prevented had the seatbelt been worn.

Plaintiff, a New Jersey resident, underwent surgery in a Connecticut hospital, during which a device was implanted in his lower spine. The device was manufactured by Defendant, a New York corporation. Following the surgery, the device's screws broke off inside Plaintiff's back, causing him extreme pain and difficulty walking. Plaintiff filed suit against the manufacturer of the device in the U.S. District Court for the Eastern District of New York. The suit, which rested on diversity jurisdiction, alleged defective design and manufacture of the device. At the same time, Plaintiff filed a complaint against the doctor and hospital with the state agency that regulates professional medical misconduct. Defendant filed a motion to dismiss Plaintiff's federal suit for failure to join necessary parties.

How should the court rule?


A. Grant the motion because of the prejudice involved in having two separate litigations.B. Deny the motion because it is not necessary for all potential joint tortfeasors to be named in a single lawsuit.C. Grant the motion and allow Plaintiff to file a new complaint without prejudice that joins the doctor and hospital.D. Deny the motion because the facts that form the basis of liability are different for the manufacturer and the doctor and hospital.

Answer choice B is correct. It has long been the rule that it is not necessary for all joint tortfeasors to be named as defendants in a single lawsuit. Tortfeasors facing joint and several liability are not parties who must be joined. Thus, answer choices A and C are incorrect. Answer choice D is incorrect because it is not the separate facts that preclude making joinder mandatory under the circumstances, but rather the fact that tortfeasors are not within the parties contemplated as indispensable under Rule 19.


While incarcerated at Elmira Prison, Defendant set off a metal detector during a routine walk-through. A search revealed that Defendant was in possession of a sharpened piece of metal, commonly referred to as a shank. The prison charged Defendant with violating standards of inmate behavior, and, following a disciplinary hearing, imposed a penalty of 120 days in a special housing unit. Subsequently, a grand jury indicted Defendant with one count of promoting prison contraband, a felony. Defendant moved to have the charge dismissed.
How should the court rule?


A. Grant the motion on double jeopardy grounds because Defendant was subjected to penalty for the same offense twice.B. Deny the motion on double jeopardy grounds because double jeopardy is not triggered by administrative action.C. Grant the motion on double jeopardy grounds because the disciplinary proceeding and penalty are tantamount to a criminal proceeding and incarceration.D. Deny the motion because the penalty imposed in the disciplinary hearing was inadequate to deter the conduct.

Answer choice B is correct. The Double Jeopardy Clause of the Fifth Amendment provides that persons shall not "be subject for the same offense to be twice put in jeopardy of life or limb." This clause prohibits multiple prosecutions for the same offense and multiple punishments for the same offense. However, the clause does not bar criminal prosecution of a prison inmate simply because the inmate was previously subjected to internal prison disciplinary action for the same conduct. Thus, answer choice A is incorrect. Prison disciplinary proceedings are civil in nature and are necessary to the safe, orderly and effective functioning of the prison. Thus, answer choice C is incorrect. Although sanctions do have a deterrent effect, that deterrent effect is aimed exclusively at deterring conduct within the prison setting, and does not constitute criminal punishment triggering double jeopardy. Thus, answer choice D is incorrect.

On December 13, 2005, Defendants signed a real estate contract to purchase Plaintiffs' home in Syracuse. The contract, which was executed by Plaintiffs on December 22, 2005, provided for a purchase price of $500,000, and contained a rider with an "attorney- approval contingency" stating as follows: "This Contract is contingent upon approval by attorneys for Seller and Purchaser by the third business day following each party's attorney's receipt of a copy of the fully executed Contract (the "Approval Period"). . . . If either party's attorney disapproves this Contract before the end of the Approval Period, it is void and the entire deposit shall be returned." After signing the contract, Defendants developed qualms about purchasing the house. They ultimately decided to buy a different residence. As a result, they instructed their attorney to disapprove the contract despite her assurances the contract was fair and reasonable. Per Defendants' instructions, the attorney did so within the three-day period allowed for invoking the attorney-approval contingency. The house remained on the market for more than three years until it was eventually sold for $350,000. Shortly thereafter, Plaintiffs sued Defendants for breach of contract.
How should the court rule?


A. For Plaintiffs, because the exercise of the contingency clause was not in good faith.B. For Plaintiffs, because the terms of the contingency were not complied with.C. For Plaintiffs, because attorney-approval contingencies are void as violating public policy.D. For Defendants.

Answer choice D is correct. Attorney-approval contingencies are routinely included in real estate contracts in upstate New York. Requiring a real estate contract to be "subject to" or "contingent upon" the approval of attorneys for both contracting parties ensures that real estate brokers avoid the unauthorized practice of law, and allows both contracting parties to have agents representing their respective legal interests. Thus, answer choice C is incorrect. If a real estate contract states it is "subject to" or "contingent upon" the approval of each party's attorney, this language means what it says: no vested rights are created by the contract prior to the expiration of the contingency period. Here, the contract explicitly stated it was contingent upon approval by attorneys for the seller and the purchaser and further provided that, if either party's attorney disapproved the contract before the end of the approval period, it would be void. There is no implied limitation that limits the circumstance under which an attorney may approve or disapprove a real estate contract pursuant to an attorney-approval contingency. Such an argument would misconstrue the implied covenant of good faith and fair dealing under New York law. Thus, answer choices A and B are incorrect. The plain language of the contract in this case makes clear that the contract was contingent on attorne

Plaintiff sold his food store business to Owner in return for a $30,000 promissory note. To secure payment, Owner and Plaintiff executed a security agreement collateralized by the chattels, fixtures, equipment, merchandise, and lease for the store. Two weeks later, Plaintiff filed a financing statement reciting the parties' names with the secretary of state that described the secured collateral as "certain food store located at 123 Main Street, Small Town, NY." Subsequently, Defendant extended $100,000 in credit to Owner for supplies and executed its own security agreement and promissory note with Owner 11 months later. Defendant also filed a financing statement reciting the parties' names with the secretary of state, describing the secured collateral as "all inventory, equipment, machinery, furniture, and fixtures, as well as accounts, contract rights, leaseholds, and general intangibles, presently owned or existing or hereafter acquired created or arising from any source whatsoever, together with any addition and accessions thereto or replacements or substitutions thereof." Thereafter, Owner defaulted in payments to Plaintiff and Defendant. Defendant foreclosed on Owner's business, auctioning the lease and contents of the store. Plaintiff filed suit, claiming that his priority interest had been violated.

Does Plaintiff or Defendant have a priority interest in the food-store collateral?


A. Plaintiff has a priority interest because he filed a financing statement that provided Defendant with adequate notice of a possible encumbrance.B. Defendant has a priority interest because Plaintiff's filing was defective and did not attach the property.C. Plaintiff has a priority interest because his security interest attached and was perfected.D. Defendant has a priority interest because it instituted the foreclosure

Answer choice B is correct. A financing statement must contain the name of the debtor, the name of the secured party or his representative, and the collateral covered by the financing statement. As long as a financing statement indicates the collateral claimed to be covered and the aforementioned names, it is deemed sufficient to provide a person with enough information to make further inquiries as to the existence and terms of the security interest. Plaintiff's filing is inadequate, and thus answer choices A and C are incorrect. There must be some designation in the financing statement, even in the most general terms, of the type of collateral covered by the security agreement. The financing statement filed on behalf of Plaintiff described the collateral as "certain food store located at 123 Main Street, Small Town, NY." In this case, there was no indication as to the assets or property covered in the financing statement. As such, it is simply inadequate as a matter of law. Answer choice D is incorrect because the determinative issue is proper filing rather than the institution of

On June 1, Plaintiff and Defendant entered into a contract for the sale of a parcel of real property in Orange County. Plaintiff provided 10% of the $500,000 purchase price upon signing the contract. The contract called for an additional $225,000 to be paid on or before July 1 and the remainder at closing. Defendant was two weeks late in making the $225,000 payment but was able to show that he was ready, willing, and able to complete the purchase at closing. Plaintiff brought an action claiming breach of contract entitling her to retain the down payment as damages, while Defendant claimed a right to specific performance.

How should the court rule?


A. For DefendantB. For Plaintiff, because Defendant failed to comply with the terms of contract as agreed to by the parties.C. For Defendant, in the amount of the return of his down payment, but the late payment precludes his ability to obtain specific performance.D. For Plaintiff, with damages in the amount of actual losses resulting from Defendant's late payment, but not necessarily the entire down payment.

Answer choice A is correct. Whether the late installment payment constitutes a material breach depends on whether time was of the essence with respect to that payment. Unless it was, Defendant had a reasonable time in which to tender performance after the July 1 date. In a real property contract, time is not ordinarily of the essence unless the agreement so provides. Here, the contract obligated Defendant to pay $225,000 on or before July 1. That language alone does not make time of the essence. Accordingly, answer choices B and D are incorrect. Defendant did not commit a material breach of the contract and is entitled to a return of his down payment. However, Defendant may obtain specific performance if he has shown that he was ready, willing, and able to fulfill his contractual obligations. Thus, answer choice C is incorrect. Note that whether specific performance is granted will depend on whether Defendant was capable of performing, and it will not turn on the fact that a late installment payment was made.

Company and Corporation entered into a 25-year contract, whereby Company undertook to supply Corporation with all of its requisite supplies. During the course of the contract term, Corporation began to believe that Company would be incapable of meeting its obligations under the agreement and demanded adequate assurances. Company was financially stable. May Corporation demand adequate assurances absent a risk of financial instability?


A. No, a demand for adequate assurances is not available because Company was not at risk of insolvency.B. Yes, because a demand for adequate assurances of future performance is always available.C. Yes, provided there are reasonable grounds for concern of risk of material breach.D. No, unless there has been an anticipatory repudiation.

Correct: Answer choice C is correct. A party to a sale-of-goods contract may demand in writing adequate assurance of future performance from the other party if reasonable grounds for insecurity exist. This is not an absolute right, thus answer choice B is incorrect. Answer choice A is incorrect because the rule requires only reasonable grounds for belief, not actual risk of insolvency. Answer choice D is incorrect because the purpose behind UCC § 2-609 is to resolve differences without the need for judicial intervention and prior to repudiation.

Petitioner District Attorney received a complaint about a criminal judge's conduct. In turn, prior to conducting any inquiry or investigation, Petitioner alerted the media and publicly disseminated a letter she wrote to the state Task Force on Criminal Courts. Specifically, the letter accused the judge of having asked the victim of an alleged sexual assault to get down on hands and knees to demonstrate the position she was forced to take when she was sexually assaulted. The criminal judge disputed the truth of the accusations, and an investigation concluded that the accusations were not supported by the evidence.

Did Petitioner's conduct violate the Rules of Professional Conduct?


A. Yes, because Petitioner's conduct demonstrated an unfitness to practice law.B. No, as long as Petitioner did not know or have reason to know the accusations were false.C. Yes, because Petitioner's misconduct violated the special responsibilities for prosecutors.D. No, because Petitioner did not violate a specific rule.

Answer choice A is correct. It is professional misconduct for a lawyer to engage in conduct that adversely reflects on her fitness to practice law. The guiding principle is whether a reasonable attorney would have notice of what conduct is proscribed. Petitioner released to the media an unwarranted false allegation that served to bring the bench and bar into disrepute. Therefore, Petitioner's conduct provided grounds for disciplinary action. Answer choice B is incorrect because it would immunize all accusations, however reckless or irresponsible, from censure if the attorney uttering them did not doubt their truth. Such a standard would be at odds with the policy underlying the Rules of Professional Conduct, which seek to establish a "minimum level of conduct below which no lawyer can fall without being subject to disciplinary action." Answer choice C is incorrect because the special responsibilities of prosecutors focus primarily on a prosecutor's duties to defendants and the public in prosecuting crimes, rather than fitness to practice law. Answer choice D is incorrect because Petitioner's public dissemination of an accusation of improper judicial conduct reflected adversely on her fitness to practice law.

Decedent asked her long-time Attorney to prepare her will and designate himself as major beneficiary. Attorney declined and suggested Decedent contact the Bar Association for the name of a lawyer to prepare the will. Attorney subsequently sent Decedent a four-page memo confirming his inability, for ethical reasons, to draft a will in which he was a major beneficiary. The memo also reviewed Decedent's assets, discussed tax and administration expenses, listed potential beneficiaries, including himself, and suggested that she consider leaving her sister a substantial bequest despite a dispute the two women were then having. The lawyer retained by Decedent drafted the will in accordance with the memo’s instructions.

Did Attorney unduly influence Decedent in violation of the Rules of Professional Conduct?


A. Yes, there is an inference of undue influence which Attorney has the burden of disproving.B. Whether or not Attorney exercised undue influence is a triable issue of fact.C. No, there is no undue influence because Decedent had the benefit of independent counsel.D. Yes, the provision of a gift under a will to Attorney creates an irrebuttable finding of undue influence.

Answer choice B is correct. A lawyer must not solicit any gift from a client, including a testamentary gift, for the benefit of the lawyer or his relatives. However, a testator’s freedom to bequeath property in accordance with her wishes should not be diminished merely because the object of the testator's generosity happens to be an attorney with whom the testator has enjoyed a beneficial professional relationship. Answer choice D is incorrect because there is a rebuttable presumption of such undue influence that is to be decided by the trier of fact. An inference of undue influence exists only when the attorney has drafted a will in which he is a beneficiary. Thus, answer choice A is incorrect. Answer choice C is incorrect because, despite an independent attorney having drafted the will, there are circumstances described in the facts that could lead a trier of fact to conclude that Decedent was unduly influenced by Attorney.

Plaintiff was a professor at a prestigious private university, University. In 2004, four female students filed sexual harassment claims against him. After reviewing the complaints, University terminated his employment. Plaintiff sued University for breach of his employment contract on the grounds that it violated its own rules and procedures for resolving sexual harassment claims by summarily firing him without an opportunity to address the accusations. Specifically, the lawsuit claimed that it was breach of contract for University not to follow procedures it had promulgated and used for resolution of sexual harassment claims brought by students against other professors.

How should the court rule?


A. For Plaintiff, because University rules and procedures are implied terms of the contract.B. For Plaintiff, because he detrimentally relied on the procedures.C. For University, because absent an express provision within the contract, failure to follow internal rules does not amount to breach of contract.D. For University, because Plaintiff's own violation of University rules and procedures amounted to breach of contract, thereby relieving University of further obligation to retain Plaintiff under the terms of its employment contract.

Answer choice C is correct. University cannot be held to have contractually bound itself to internal rules when it hired Plaintiff. It defies reason to consider that Plaintiff could argue that he detrimentally relied on University's procedures for dealing with sexual harassment claims in accepting his position. Thus, answer choices A and B are incorrect. An implied-in-fact contract would arise from a mutual agreement and intent to promise, when the agreement and promise have simply not been expressed in words. This type of contract still requires consideration, mutual assent, legal capacity, and legal subject matter. The conduct of a party may manifest assent if the party intends to engage in such conduct and knows that such conduct gives rise to an inference of assent. Thus, a promise may be implied when a court may justifiably infer that the promise would have been explicitly made, had attention been drawn to it. Although employer handbooks generally contain information concerning rights and responsibilities, the concept of handbooks as part of a contract, with commitments and expectations on both sides, is not universally accepted. University's sexual harassment policy and its procedures for dealing with sexual harassment claims are not essential to a professor's employment duties, and the rights are only indirectly affected by these provisions. Therefore, University's rules and procedures concerning sexual harassment cannot be said to be deemed a part of the employment contract with the professor. Thus, answer choice D is incorrect.


Next Question

One evening, a team of police officers with extensive drug arrest experience observed a transaction between Defendant and a known drug dealer, which had all the indicia of an illegal drug transaction. Defendant was taken into police custody and transported to a police station where his clothing was searched, but no drugs were found. Police Officer placed Defendant in a private detention cell and asked him to remove his clothing. Defendant was ordered to bend over or squat, at which point Officer observed a string or piece of plastic hanging out of Defendant's rectum. Believing that the string was attached to a package of drugs hidden inside defendant's body, Officer ordered Defendant to remove the object. When Defendant refused, Officer proceeded to hold Defendant while another officer pulled on the string and removed a plastic bag found to contain crack cocaine. After being charged with criminal possession, Defendant moved to suppress the crack cocaine found on his body.
How should a court rule?


A. Grant the motion, because the officers failed to procure a warrant to search Defendant's body.B. Grant the motion, because there were insufficient indicia of exigent circumstances that would excuse obtaining a warrant.C. Deny the motion, because body-cavity searches are routine in drug arrests.D. Deny the motion, because there were specific articulable facts to justify the intrusion.

Answer choice D is correct. Visual body cavity examinations of detainees following visitations with persons from outside the facility are permitted. Such visual cavity searches of pretrial detainees can be conducted on less than probable cause grounds, so long as they are conducted in a reasonable manner. Visual cavity inspections and manual body-cavity searches cannot be routinely undertaken as incident to all drug arrests or permitted under a police department's blanket policy that subjects persons suspected of certain crimes to these procedures. Thus, answer choice C is incorrect. There must be particular, individualized facts known to the police that justify subjecting an arrestee to these procedures that, along with any logical deductions, reasonably prompt the intrusion. In addition, the reasonableness of the manner in which the search is conducted should be evaluated by reference to where, how, and by whom the inspection occurred (e.g., usually in a private location, by a person of the same gender and without causing the arrestee to suffer further undue humiliation). Here, the facts support the conclusion that the strip search and visual cavity inspection of Defendant's body were constitutionally valid and reasonable and were not subject to the requirement of a warrant or exception to the warrant requirement. Thus, answer choices A and B are incorrect.

Testator left a will, in which a testamentary trust was created. The trust paid income to Petitioner for life, and upon Petitioner's death, Petitioner had the power to appoint a beneficiary of the principal. The trust further provided that, in the event Petitioner failed to exercise the power of appointment, the principal was to vest equally in Petitioner's three infant children. During his lifetime, Petitioner elected to terminate the trust and gave Trustee written notice to that effect on his own behalf and on behalf of his children.

What effect did Petitioner's notice have on the trust?


A. The trust is presumptively irrevocable and cannot be revoked.B. Although the trust is presumptively irrevocable, it can be revoked with the written consent of all beneficiaries.C. A trust with a minor beneficiary is incapable of being revoked because a minor is incapable of giving consent to revocation.D. Petitioner's power of appointment vests within him the power and authority to revoke the trust.

Answer choice C is correct. The law permits the revocation of trusts, even irrevocable trusts, upon the written consent of all persons beneficially interested. Thus, answer choice A is incorrect. Answer choice B is incorrect because such revocation cannot be accomplished if there are infants or children who have a beneficial interest in the trust. Neither an infant nor a guardian appointed to represent his interests is capable of giving consent on behalf of that child. Answer choice D is incorrect because a power of appointment vests in the holder the power to appoint beneficiaries of a trust, but absent an express statement to the contrary within the trust instrument, it confers no power of revocation.


Defendant hired Plaintiff to work as a financial services representative. Several years later, Defendant terminated Plaintiff's employment following an audit that revealed irregularities. Upon termination of a registered representative, the NASD requires member-firms to complete and file a Uniform Termination Notice for Securities Industry Registration within 30 days of dismissal and to provide a copy of the form to the employee. Consistent with these requirements, Defendant prepared the form, and included the statement that "AN INTERNAL REVIEW DISCLOSED PLAINTIFF VIOLATED COMPANY POLICIES AND WAS A POSSIBLE ACCESSORY TO MONEY LAUNDERING VIOLATIONS." Plaintiff sued Defendant for defamation. Defendant moved for summary judgment dismissing the complaint in its entirety.
How should the court decide?


A. For Defendant, because the statement was absolutely privilegedB. For Plaintiff, provided the statement is falseC. For Plaintiff, because Defendant had no more than a qualified privilege in the statementD. For Defendant, unless the statement was made with malice

Answer choice A is correct. Public policy mandates that certain communications, although defamatory, cannot serve as the basis for the imposition of liability in a defamation action. When there is a compelling public policy, a speaker will not face liability for defamation; the law affords an absolute privilege. An absolute privilege immunizes a communicant from liability in a defamation action. The absolute privilege generally is reserved for communications made by individuals participating in a public function, such as executive, legislative, judicial or quasi-judicial proceedings. This protection is designed to ensure that such persons' own personal interests - especially fear of a civil action, whether successful or otherwise - do not have an adverse impact upon the discharge of their public function. A comparable public purpose is served by the NASD's regulatory regime for the securities industry. As a securities association, it is a quasi-governmental entity delegated with authority to enforce the requirements of the Securities and Exchange Act and is the primary regulator of the broker-dealer industry. Thus, statements made by an employer on the form should be subject to an absolute privilege. Answer choices B, C and D are incorrect.

In an action for wrongful death arising out of a single-vehicle car accident in New York, Plaintiffs are New Jersey domiciliaries, as was their daughter. Their daughter was a passenger in the vehicle and was one of three teenagers killed in the accident. Defendant, a Texas resident, leased the car in New Jersey. Defendant's son, a New Jersey domiciliary, was the driver of the car and was also killed in the accident. The second passenger killed in the accident was a New York domiciliary. New Jersey law does not impose vicarious liability on vehicle owners or lessors except in limited situations not present here. New York law recognizes such vicarious liability. If New Jersey law were to apply, Plaintiffs would have no cause of action. Which jurisdiction's law should apply?


A. New York because the accident occurred in New York.B. New Jersey because the state has the greatest interest in the outcome of the litigation.C. New Jersey because both a plaintiff and a defendant to the action resided in New Jersey.D. New York because New Jersey law provides no remedy for the defendant.

Answer choice A is correct. New York law governs. Vicarious liability is a rule of loss allocation that is subject to one of the three choice-of-law rules. In a split-domicile case, if the interested parties do not share a common domicile, then the governing law will be that of the place where the accident occurred, unless there are good substantive-law reasons not to apply that law. In this case, Plaintiffs are from New Jersey and Defendant is from Texas, so this is a split-domicile case. The site of the accident was New York, and there is no indication that the case should not be tried there. Thus, answer choices B, C, and D are incorrect. The driver and the second passenger are not parties, so their domiciles do not matter.

Approximately 1,000 grain suppliers from 14 states filed a class-action suit against Bread Corporation in the U.S. District Court for the Northern District of New York. The suppliers alleged a scheme by which Bread Corporation rigged scales weighing grain supplied so as to underreport the weight of grain sold and paid for, thereby resulting in underpayment to each supplier. The amount of damages suffered by each of the suppliers varied; most were less than $25,000, one supplier claimed to have lost exactly $75,000, and two suppliers had losses exceeding $80,000. Plaintiff conceded that there was diversity but objected to the action being in federal court based upon the fact that most of the individual claims were less than $25,000.

How should the court rule?


A. The amount in controversy meets the threshold because at least one member of the class has a claim that amounts to greater than $75,000.B. Only the individuals with the $75,000 claim and the claims in excess of $80,000 may proceed in federal court; the remaining claims must be severed and remanded to state court.C. The amount in controversy meets the threshold because the collective amount claimed by the class exceeds $75,000.D. Only the individuals with the claims exceeding $80,000 may proceed in federal court; the remaining claims must be severed and remanded to state court.

Answer choice A is correct. If the other elements of jurisdiction are present and at least one named plaintiff in the action satisfies the amount-in-controversy requirement, then the other plaintiffs may be made part of the class under the doctrine of supplemental jurisdiction. Thus, answer choice C is incorrect. If a well-pleaded complaint has at least one claim satisfying the amount-in-controversy requirement and there are no other relevant jurisdictional defects, then the district court has original jurisdiction over that claim. A court with original jurisdiction over a single claim has original jurisdiction over a civil action, even if that action comprises fewer claims than were included in the complaint. Once a court has original jurisdiction over the action, it can then decide whether it has a constitutional and statutory basis for exercising supplemental jurisdiction over other claims in the action. Section 1367(b) does not withdraw supplemental jurisdiction over the claims of the additional parties in this fact pattern. The "indivisibility theory" - that all claims must stand or fall as a single, indivisible action - is inconsistent with the whole notion of supplemental jurisdiction and is in contradiction with the practice of allowing federal courts to cure jurisdictional defects by dismissing the offending parties instead of the entire action. Thus, answer choices B and D are incorrect.

12. (Question ID#3782)


Which of the following causes of action is not subject to the one-year statute of limitations?


A. Action to recover a chattel or damages for the taking of a chattelB. Misappropriation of a plaintiff’s picture or nameC. Action by tenant for damages for retaliatory evictionD. Action upon an arbitration award

Answer choice A is correct. Actions to recover a chattel or damages for the taking of a chattel are subject to a three-year statute of limitations. Answer choices B, C and D are incorrect, as each of those causes of action are subject to a one-year statute of limitations.


Plaintiff advanced $300,000 to Co-op, a five-unit residential building. Plaintiff made the payment with the understanding that she would acquire the proprietary lease and shares entitling her to possess the garden apartment within Co-op. Thereafter, Plaintiff advised Co-op that she intended to renovate the apartment, and Co-op demanded an additional $50,000 and other concessions as conditions for allowing her to renovate the garden unit. Plaintiff refused, instituted a lawsuit, and filed a financing statement against Co-op listing the covered collateral as: "All right, title, and interest of debtor Co-op as lessor and/or seller in and to the use, occupancy, and the proprietary leasing of and any proprietary lease and shares issued or to be issued by debtor appurtenant to the garden apartment at Co-op and any proceeds of the foregoing." Co-op objected to the filing of the financing statement.

Did Plaintiff properly file a financing statement to secure her interest?


A. Yes, Plaintiff properly filed a financing statement to put potential third-party buyers on notice of her claims to the proceeds.B. Yes, Plaintiff properly filed a financing statement because the parties were on notice of the advance, thereby creating a common-law interest in proceeds of any sale, transfer, or encumbrance of the apartment.C. No, Plaintiff is not entitled to file a financing statement absent the authorization of Co-op as debtor.D. No, Plaintiff is not entitled to file a financing statement to secure an interest in shares of a cooperative.

Answer choice C is correct. A person may file a financing statement if the debtor authorizes such filing in an authenticated record. There must be some written evidence of the debtor's intent to grant the security interest. Here, Plaintiff did not receive authorization from Co-op before filing a financing statement. In addition, a person may file a financing statement if there is a security agreement that creates a security interest as predicate to the filing of a financing. A security agreement is one that creates or provides for a security interest. Here, there is no mention of a security agreement between Plaintiff and Co-op, and thus answer choices A and B are incorrect. Answer choice D is incorrect because there is nothing in Article 9 that precludes filing a financing statement to secure an interest in cooperative shares.

Plaintiff, a Colorado domiciliary, commenced this action in a New York court seeking damages for injuries she sustained in a boating accident on a lake in Ontario, Canada. Defendant 1, who was driving the boat that Plaintiff was in, is a domiciliary of New York. At the time of the accident, Defendant 2, whose jet ski collided with Plaintiff, was a domiciliary of Alabama. Which law should apply?


A. Canadian law.B. New York or Alabama law, because U.S. courts will not apply foreign law.C. Colorado law.D. New York law because the action was commenced in a New York court.

Answer choice A is correct. If conflicting tort laws relate to the allocation of losses (rather than to the regulation of conduct), then the courts will look to the domiciles of the parties. In this case, the domiciles are split, so the court will apply the split-domicile rule--the law of the place where the tort occurred will apply. The tort occurred in Canada, so Canadian law will apply, making answer choices B, C, and D incorrect. Note also that answer choices B and D are misstatements of the law.

A corporation incorporated in New York had its principal office in Buffalo. The president of the corporation owned 51% of the shares of the corporation. The vice president of the corporation owned 35% of the shares. S was a shareholder who owned 5% of the shares but did not have a position as an officer of the corporation. A buyer approached the president with an offer to purchase her shares, along with an additional bonus upon her stepping down from her office as president. The president has approached an attorney to determine whether the agreement to sell her shares is valid.
How should the attorney advise the president of the corporation?


A. The agreement breaches the president's duty to minority shareholders not to take advantage of her majority position.B. The agreement is valid because shareholders have no duties to one another.C. The agreement breaches the president's fiduciary duty as officer to the corporation.D. The agreement is valid because an officer is not subject to any fiduciary duties to its corporation.

Answer choice C is correct. The contract to resign her position as president in return for a bonus violates an officer's duty to the corporation not to sell an asset of the corporation for her own benefit. Thus, answer choice C is correct and answer choice D is incorrect. Shareholders in a control position owe a fiduciary duty of good faith to the minority shareholders. Thus, answer choice B is incorrect. Such duty arises when the controlling shareholder is (i) selling her interest to an outsider, (ii) seeking to eliminate other shareholders from the corporation, or (iii) receiving a distribution denied to the other shareholders. However, the duty is not breached simply because a majority shareholder is afforded an opportunity to sell her shares as a majority shareholder. Thus, answer choice A is also incorrect.

what type of murder in NY has the affirmative defense of extreme emotional disturbance?



what are the three types of murder in the second degree in NY?

N.Y. Penal Law § 125.25 defines three types of murder in the second degree: intentional murder, depraved-indifference murder, and felony murder. Only N.Y. Penal Law § 125.25(1), dealing with intentional murder, contains a provision for mitigation of the charge by extreme emotional disturbance.

Plaintiffs were employees of a closely held corporation organized under the laws of New York. Because of an economic downturn, the corporation fell behind on paying Plaintiffs their salaries before filing for bankruptcy. A owned 51% of the shares, B owned 25% of the shares, and C and D each owned 12% of the shares. A, B, and C were directors of the corporation, and A and B were president and vice president of the corporation. Plaintiffs commenced an action against A, B, C, and D individually for their unpaid salaries. Defendants moved to dismiss. How should the court rule?


A. For Defendants because individual shareholders, officers, or directors are not individually liable for the debts of the corporation.B. Dismiss the actions against B, C, and D because only a majority shareholder can be personally liable for employee wages.C. For Plaintiffs because, under the circumstances, New York holds shareholders individually liable for wages due employees.D. For Plaintiffs provided they are successful in piercing the corporate veil.

Answer choice C is correct. The 10 largest shareholders of a close corporation are personally liable for all debts, wages, or salaries due and owing to any of its laborers, servants, or employees other than contractors for services performed by them for such corporation. Thus, answer choices A, B, and D are incorrect. Answer choice D is further incorrect because the corporate veil need only be pierced if it is necessary to prevent inequity; in this case, it is unnecessary to pierce the corporate veil because the shareholders are personally liable for salaries.


S


eller placed her apartment for sale and hired a real-estate broker, Agent, to procure a buyer. Seller then confided to Agent that Seller preferred to keep the apartment and purchase the adjacent apartment and combine the two into a single unit. Thereafter, Agent made an offer on the apartment herself, and the parties entered into a written contract for its sale. After entering into the contract, but prior to closing, Agent learned that the adjacent apartment had been placed on the market. Agent did not disclose this fact to Seller and subsequently purchased the adjacent apartment, in addition to Seller's apartment.
Will Seller succeed in an action for breach of fiduciary duty for Agent's failure to advise Seller that the adjacent apartment was for sale?


A. Yes, because a broker has a duty to disclose to the principal the nature and extent of the broker's interest in the transaction.B. No, because the broker/principal relationship and accompanying fiduciary duty was completed before Agent learned that the neighbor's apartment was on the market.C. Yes, because a real-estate agent is subject to an undivided duty of loyalty until the closing of the sale.D. No, because Agent's duty to Seller is limited to finding a ready, willing, and able purchaser.

Answer choice B is correct. When a broker's interests or loyalties are divided because of a personal stake in the transaction, the broker must disclose to the principal the nature and extent of the broker's interest in the transaction or the material facts illuminating the broker's divided loyalties. Here, the broker/principal relationship and accompanying fiduciary duties to deal honestly, fairly, and in good faith, and to disclose all known facts that materially affect the contract property, were severed upon the parties entering into the contract and before Agent learned that the neighbor's apartment was for sale. Thus, answer choices A and C are incorrect. Answer choice D is incorrect because, although the contract between Seller and Agent correctly recites a real-estate broker's obligation, it does not address the agent's fiduciary duties of loyalty, obedience, and reasonable care.

Plaintiff was an insurance agent for Defendant Insurance Company. As part of their arrangement, Defendant agreed to lease to Plaintiff computer hardware and software to facilitate her collection and transfer of customer information. In addition to the entry of business data, Plaintiff also used the computer system for personal data that pertained to her customers. On a daily basis, Defendant would upload all of the information from Plaintiff's system to its centralized computers. Several years into the relationship, Plaintiff received a letter from Defendant that their relationship was being terminated. The next day, Defendant repossessed its computer system and denied Plaintiff further access to the computers, electronic records, and data. Consequently, Plaintiff was unable to retrieve her personal information stored on the computers. Plaintiff instituted an action against Defendant for conversion of her personal information. Defendant moved to have the cause of action dismissed.
How should the court decide?


A. For Defendant, because a conversion action is only available for tangible propertyB. For Defendant, because it owned the computer system on which the personal information was storedC. For Plaintiff, unless Defendant demonstrates the information is inextricably merged with the softwareD. For Plaintiff, because Plaintiff is entitled to recovery for misappropriation of her property irrespective of whether it is in tangible or intangible form

Answer choice D is correct. Although historically an action for conversion would not normally lie when it involves intangible property because there is no physical item that can be misappropriated, more recently, courts have concluded that a plaintiff can maintain a cause of action for conversion when the defendant infringes on the plaintiff's intangible property right to a musical performance by misappropriating a master recording; a tangible item of property capable of being physically taken. Thus, answer choice A is incorrect. A document stored on a computer hard drive has the same value as a paper document kept in a file cabinet. Furthermore, it generally is not the physical nature of a document that determines its worth, it is the information memorialized in the document that has intrinsic value, which makes answer choices B and C incorrect.

* Plaintiff and Defendant worked together to train Defendant for the snowboarding competition in the Olympics. After Defendant won the gold medal, Defendant and Plaintiff agreed that Plaintiff would continue to coach Defendant for as long as Defendant snowboarded professionally. Thereafter, in an interview broadcast nationally, Defendant stated, "Plaintiff will be my trainer for as long as I am a professional snowboarder." After six months, Plaintiff and Defendant's relationship deteriorated, and Defendant terminated Plaintiff's employment. Will Plaintiff be successful in a suit for breach of contract?



* No, because the agreement was for an indefinite duration, and thus was an employment-at-will contract terminable at any time.
* Yes, because the length of time that Defendant is capable of being professional is an ascertainable period, and the contract was breached when terminated beforehand.
* No, because the contract violates the Statute of Frauds.
* Yes, because the contract was memorialized in a recorded interview, and thus, a writing is not required.

Answer choice B is correct.



The important rules:



In NY, an employment relationship is presumed to be a hiring at will unless there is an agreement establishing a fixed duration (not necessarily specific time frame, but an end point that would be ascertainable to the parties and the court - like for the rest of my professional snowboarding career).



Also note: the SOL would apply to this contract if it were certain that the snowboarding career would last more than one year. Because it is not, the agreement does not have to be in writing.



* Plaintiff entered into a professional services contract with a lawyer to represent Plaintiff in a real estate transaction. The lawyer was negligent in his representation of Plaintiff, and Plaintiff brought a legal malpractice claim against the lawyer for failure to exercise due care in the performance of a contract. Plaintiff sought financial compensation. What is the statute of limitations for Plaintiff to bring a viable claim?

* Six years from the date of contract.
* Three years from the date of the lawyer's negligent conduct.
* Two and a half years from the date Plaintiff terminated the professional relationship with the lawyer.
* Six years from the date of the alleged wrongdoing.

Answer choice D is correct: For breach of contract, six years from the date of the wrongdoing. The choice of the applicable statute of limitations relates to the remedy sought rather than the theory of liability. In applying the statute of limitations, the "reality" or "essence" of the action should be examined, not its form. The lawyer as Defendant would argue the "essence" of the malpractice action is negligence. However, a legal malpractice claim in which the remedy sought is damages relating solely to a plaintiff's pecuniary or property loss and which arises out of the contractual relationship is a breach of contract action. Therefore, answer choice B is incorrect. Answer choice C is incorrect because the two and a half year statute of limitations for professional malpractice is limited in its application to medical malpractice claims only. Answer choice A is incorrect because the statute of limitations begins to run from the date of the breach rather than the date of the contract.

* Plaintiff, a New York corporation, filed suit against Defendant alleging trademark infringement under the Federal Trademark Act in the U.S. District Court for the Southern District of New York. Plaintiff is the manufacturer of a doll with a distinctive look, which includes curly green hair and pink eyes, known as the "Snuggle." Defendant, a Hawaii corporation with its principal place of business in Honolulu, operates a website targeted at children ages five through nine. Defendant obtained the exclusive rights to use domain names "snuggle.com," "snuggle.net," and "snugglenews.com" on the Internet. Defendant's website contains information about the doll and games featuring a character bearing a strong resemblance to the Snuggle doll. Customers who want to subscribe to the site fill out an online application and are provided a password. Defendant's contacts with New York have occurred exclusively over the Internet. Defendant's offices, employees, and servers are located in Hawaii, and the company maintains no offices, employees, or agents in New York. Defendant has approximately 15,000 paying subscribers worldwide. Approximately two percent of subscribers, or 300 people, are New York residents. By motion, Defendant raised the defense that the court lacks personal jurisdiction. How should the court rule?

Answer choice D is correct. In the absence of general jurisdiction, specific jurisdiction allows a court to exercise personal jurisdiction over a non-resident defendant for forum-related activities if: (i) the non-resident defendant has sufficient minimum contacts with the forum state, (ii) the claim asserted against the defendant must arise out of those contacts, and (iii) the exercise of jurisdiction must be reasonable. Although Defendant and his agents have not entered New York, they created a website with the purpose of assisting Defendant with conducting business transactions within the state of New York. This is more than merely setting up a website and making it accessible to the public, which alone may not subject a defendant to personal jurisdiction. Thus, answer choices A and B are incorrect. Here, Defendant has not merely posted information on a website that is accessible to New York, it conducted electronic commerce by contracting with approximately 300 New York residents; thus, answer choice C is incorrect.

* In her will, Testator bequeathed real and personal property to Son, whom she gave up for adoption. Son predeceased her. Do her bequests pass to his issue under New York's anti-lapse statute?

Answer choice C is correct. For the purpose of triggering the anti-lapse statute, issue includes adopted children and their issue to the extent that they would be included in such general disposition. An adopted-out child will not take in a class gift from a birth relative unless that child is specifically named in a biological ancestor's will or the gift is expressly made to issue, including those adopted out of the family. Thus, adopted children and their issue are ordinarily strangers to their birth relatives, unless a bequest is to named adopted-out children, in which case they are no longer identified as strangers. Consequently, biological children who are not "strangers" are "issue" under the anti-lapse statute. When Testator named her adopted-out child, Son, a beneficiary of her will, she made him a nonstranger, and thus her issue, with respect to the relevant bequest. Thus, Son's issue are entitled to the benefit of the anti-lapse statute, and answer choices A, B, and D are incorrect.

* Buyer and Seller entered into a contract for Buyer to purchase Seller's home. As part of the contract of sale, Buyer agreed to take the property subject to a mortgage held by Lender, and Seller promised to continue to make payments until the mortgage was paid. Because Lender was Seller's godfather and a close family relative, Lender did not record the mortgage. After closing, Seller stopped making payments, and Lender filed suit against Buyer and Seller individually and sought foreclosure on the property. Both parties moved to have the action dismissed against them. How should the court rule?



Answer Choices: 1. Dismiss the action because Lender failed to record the mortgage.
2. Dismiss the action against Seller because Buyer took the property subject to the mortgage.
3. Dismiss the action against the Buyer personally, but allow Lender to foreclose on the property.
4. Dismiss the action against neither Buyer nor Seller personally because they were both on notice of the mortgage.

Answer choice C is correct. If the buyer takes title subject to an existing mortgage, rather than assuming the mortgage, then the buyer is not personally liable upon default; only the original debtor is personally liable. Thus, answer choices B and D are incorrect. Nonetheless, property subject to the security interest of a lender can be foreclosed, irrespective of whether that security interest was recorded or not, provided the owner of the property is on notice of that interest. Thus, answer choice A is incorrect. Recording is just one means by which a subsequent purchaser is put on notice of a creditor's security interest in real property, thereby allowing foreclosure in the event the debt is not paid. However, notice by other means makes the property equally subject to the possibility of foreclosure. In this case, Buyer was on actual notice of the mortgage because he signed a contract agreeing to be subject to that mortgage, and thus he may lose the property to foreclosure even if he is not personally liable on the debt.

* Plaintiff brought a malpractice action against her attorney. Plaintiff employed a process server who attempted to personally serve defendant Attorney by appearing at his office during business hours. Finding the door locked without anyone answering, the process server affixed the summons and complaint to Attorney's office door and followed up by mailing the summons and complaint to Attorney's office. Attorney appeared, and made a motion to dismiss for improper service. How should the court rule?



* Grant the motion because a summons and complaint in a professional malpractice case must be personally served.
* Grant the motion because one attempt at personal service is not sufficient due diligence to justify substituting affix and mail service.
* Deny the motion because service was proper.
* Deny the motion, provided the court reasonably believes the process server attempted personal service prior to affixing the papers to his door.

* Answer choice B is correct. Service on a natural person must be delivered personally to the defendant, its agent, or a person of suitable age and discretion at defendant's home or business. Only when such service cannot be accomplished with due diligence is service by affixing legal papers to the door followed by mailing permitted. Answer choice A is incorrect because there is no exception for legal malpractice cases. What constitutes due diligence is a question of fact, but courts have repeatedly held that a single visit when no one is home is insufficient. Thus, answer choices C and D are incorrect.
* Subject Outline: NEW YORK PRACTICE AND PROCEDURE (CPLR)

* Plaintiff subcontractor and Defendant contractor entered into an agreement whereby Plaintiff would provide electrical installation work for Defendant contractor's construction projects. The parties' agreement stated that Defendant contractor would pay Plaintiff subcontractor 1% of the money received for the job, once Defendant was paid by a customer. After the project was completed, the customer declared bankruptcy and defaulted on paying Defendant. In turn, Defendant refused to make payment to Plaintiff, and Plaintiff sued for non-payment. How should the court rule?

Answer choice A is correct. The New York Court of Appeals has held that "pay-if-paid" contracts violate New York public policy and that "pay-if-paid" clauses are to be interpreted to mean payment is due within a reasonable time. Thus, answer choices B and D are incorrect. Answer choice C is incorrect because although such provisions run counter to New York public policy, there is nothing about such a provision that makes it illegal. Further, Plaintiff is entitled to contractual relief, not just quasi-contractual relief.

* Buyer entered into an agreement with Seller to design, engineer, construct, test, and deliver three shipping vessels for the purchase price of $3,000,000. Seller completed and delivered the vessels in the summer. Buyer asserts that, at the time of delivery and after placing the vessels in operation, certain structural and mechanical problems became apparent. Buyer immediately complained to Seller, and Seller repeatedly assured Buyer that the problems would be rectified. Over time, Seller repeatedly promised and made efforts to repair the vessels before the Coast Guard ordered them removed from the water two years later. Buyer brought an action to rescind the contract, and Seller moved to dismiss. How should a court rule?


* In favor of Seller because too much time has passed.
* In favor of Buyer because Seller's breach substantially impaired the value, and he reasonably relied on Seller's promise to cure.
* In favor of Seller because Buyer failed to reject upon delivery.
* In favor of Buyer because rescission is the only available remedy.

Answer choice A is correct. Acceptance of goods occurs when a buyer fails to make an effective rejection. A buyer may revoke acceptance if a defect or nonconformity substantially impairs the value, and if a buyer accepted the goods on the reasonable assumption that the seller would cure the defect and the seller has failed to do so. In this case, Buyer admits that he accepted the vessels, but he is seeking to rescind or revoke acceptance. Answer choice B is incorrect because such revocation of acceptance must occur within a reasonable time after the buyer discovered or should have discovered the nonconformity and before any substantial change in the condition of the goods not caused by the defect. Answer choice C is incorrect because Buyer did not have to reject the goods upon delivery because Seller agreed to cure the defect. However, Buyer continued to use the goods without notifying Seller of his intent to rescind and demand return of the purchase price. Buyer's failure to effectively revoke acceptance of the vessels does not impair any other remedy provided by UCC Article 2 for nonconformity. Buyer's repeated complaints and requests for repairs were sufficient to preserve his right to sue for damages, and Buyer can seek damages for his losses resulting from Seller's breach; thus, answer choice D is incorrect.

* Petitioner worked for Respondent, a not-for-profit corporation incorporated under the laws of New York. After Petitioner had been in his position for two years, the corporation began to operate at a loss. The board of directors met and concluded that Petitioner had failed to remedy financial problems, and therefore it terminated his employment, effective immediately. Petitioner claimed that his termination did not comply with the corporation's personnel manual, because the manual stated that employees would be informed of problems in job performance and given a period of time to improve. Petitioner filed an Article 78 proceeding against the corporation for violation of its internal rules. Respondent moved to have the proceeding dismissed. How should the court rule?


* Grant the motion to dismiss on the grounds that a corporation is not a state organization subject to Article 78 proceedings.
* Grant the motion because Article 78 proceedings cannot be used to circumvent the at-will employment law doctrine.
* Grant the motion because the appropriate remedy for violation of a corporation's personnel manual is a breach of contract claim.
* Deny the motion because corporations can be compelled to fulfill obligations imposed by their internal rules.

Answer choice D is correct. Article 78 proceedings are valid against nonpublic, not-for-profit corporations. Therefore, answer choice A is incorrect. Answer choice B is incorrect because there is no such rule of law. Answer choice C is incorrect because failure to follow a personnel manual is not a colorable contract claim, though it is a valid claim under Article 78.

* Partnership was formed by Partner A, Partner B, and Partner C to hold title to a shopping center. Partner C was responsible for managing the property; Partner A and Partner B took no part in the management. After the partnership decided to sell the shopping center property, Partner C took a $10,000 fee for services rendered in negotiating the sale. Partner A and Partner B sued to recover the fee, claiming the $10,000 fee was not contemplated in their partnership agreement, and it was contrary to the agreement's language, which allowed a draw of only $1,200 annually in consideration for acts performed by Partner C relating to his property-management duties. May Partner C be paid additional fees for performing the extra service of negotiating the sale of the property, which was outside the scope of his normal partnership duties?


* No, because absent an express agreement to the contrary, partners in a partnership share compensation and liabilities equally.
* No, because a partner receiving extra compensation for providing services amounts to self-dealing and a conflict of interest.
* Yes, because a partner is entitled to be compensated for extraordinary services.
* Yes, so long as the services are provided in conjunction with winding up partnership affairs.

Answer choice C is correct. Courts have held that partners do not always have equal interests or equal liabilities for partnership business, and thus answer choice A is incorrect. Consequently, partners who do provide extraordinary services are entitled to additional compensation. Partners cannot engage in self-dealing, or otherwise put personal interests in conflict with those of the partnership. However, answer choice B is incorrect because providing a necessary service for the benefit of the partnership is not self-dealing. Further, extra compensation is permitted to partners who provide services in winding up partnership affairs, but this in no way conflicts with the principle that extraordinary services should receive extra compensation; thus, answer choice D is incorrect.

* Plaintiff, her husband, and their three-year-old daughter were residents in an apartment rented from Defendant. Plaintiff's daughter allegedly sustained injuries as a result of Defendant's negligence in failing to repair a window. During defense counsel's cross-examination of Plaintiff, he asked, "Ma'am, did you ever at any time take money that you weren't entitled to?" The defense counsel's question referred to information contained in a confession of judgment that Plaintiff had entered into with the Department of Social Services wherein she admitted that she received money improperly from the department. When Plaintiff answered in the negative, defense counsel offered the judgment as evidence. Plaintiff's counsel objected. How should the court rule?

1. The confession of judgment is inadmissible because it is irrelevant.
2. The confession of judgment is inadmissible because extrinsic documentation cannot be used to contradict a witness's testimony.
3. The confession of judgment is admissible because it challenges Plaintiff-witness's credibility.
4. The confession of judgment is admissible as evidencing moral turpitude.

Answer choice B is correct. In New York, the general rule is that a witness may be cross-examined with respect to specific immoral, vicious, or criminal acts that have a bearing on the witness's credibility. To be relevant on credibility, the inquiry must have some tendency to show moral turpitude. Whether or not Plaintiff's receipt of funds from the Department of Social Services was a proper subject for cross-examination is debatable, and thus answer choice A is incorrect. However, the matter was unquestionably collateral. It was neither relevant to some issue in the case other than credibility nor was proof of it independently admissible to impeach a witness. Unlike material facts in dispute -- or matters such as a witness's bias, hostility, or impaired ability to perceive, which may be proved independently for impeachment -- Plaintiff's alleged prior misconduct had no direct bearing on any issue in the case other than credibility. If proven, it would show only that Plaintiff had acted deceitfully on a prior unrelated occasion. The matter was, therefore, collateral and, under the settled rule, could not be pursued by the cross-examiner with extrinsic evidence to refute Plaintiff's denial. Thus, both answer choices C and D are incorrect.

* In a contested administration proceeding, Petitioner sought letters of administration in Decedent's (his paternal first cousin) estate. Petitioner and Brother claimed that they were the sole distributees of the estate, that they had full knowledge of Decedent's family tree, and that there were no surviving distributees on the maternal side of Decedent's family. Public Administrator filed objections and claimed a superior prior right to letters of administration. How should the court rule?


* For Petitioner because absent a spouse, issue, parents, or grandparents, he and Brother are next of kin.
* For Public Administrator because absent a spouse, issue, parents, or grandparents, Public Administrator is required as a matter of law to administer the estate.
* For Petitioner because there is a preference for appointing family members over Public Administrator to administer an estate.
* For Public Administrator because the distributees are only on the paternal side of Decedent’s family tree.

Answer choice D is correct. If a decedent's distributees are grandparents or their issue, then the decedent's estate shall be distributed one-half to the maternal side and one-half to the paternal side. N.Y. Surrogate's Court Procedure Act § 1001 provides for the appointment of a public administrator to ensure that a diligent search is made to locate all distributees on both sides of the family in circumstances in which the decedent is survived by cousins on only one side of the family. Thus, the statute appears to mandate the appointment of a public administrator in circumstances such as this proceeding, when the only known distributees are first cousins on one side of Decedent's family. Thus, answer choices A, B, and C are incorrect.

* In 2003, Testator created, by will, a trust that provided that income should be payable to Wife for the remainder of her life and that Wife should appoint any grandchild or grandchildren to take the remainder upon her death. In 2007, Grandchild A had been born. In 2008, Testator died. In 2010, Wife died, and her will provided that Grandchild A should inherit the principal of Testator's trust when she reaches the age of 35. Is the gift valid?

Answer choice A is correct. The Rule Against Perpetuities holds that a trust may fail if an interest thereunder does not vest within a life in being plus 21 years. The period begins to run at the time the trust is created. A trust created by will is created as of the date of death of the testator rather than the date the will is executed. Here, Testator died in 2008. In 2007, Grandchild A had been born. Consequently, irrespective of what age her interest in the principal vests, she will have inherited it within a life in being plus 21 years. Thus, answer choices B and C are incorrect. Answer choice D is incorrect because a violation of the suspension rule occurs when the power to convey is suspended for greater than a life in being plus 21 years. Here, because Grandchild A will have inherited the principal within her lifetime, she will have the power to convey the same, and thus there is no suspension rule violation.

* Plaintiff, a businessman domiciled in Texas, lent $200,000 to the ABC Trust. ABC Trust was organized under the laws of Texas and held title to various real-estate investments in and around Texas for the benefit of its beneficiaries. Trustee is from New York, and the beneficiaries are three siblings, two of whom reside in Texas and one of whom resides in New York. After ABC Trust defaulted in repaying the loan, Plaintiff filed suit. Can the suit be filed in federal court in New York?


* Yes, because there is diversity between Plaintiff and at least one beneficiary.
* No, because Plaintiff and one of the beneficiaries are from Texas.
* Yes, because there is diversity between Plaintiff and Trustee.
* No, because Texas has the greatest interest in the outcome of the litigation.

Answer choice C is correct. It is the trustee who is the party in interest because it is the trustee that has the power to manage and control the trust, borrow money on behalf of the trust, and file suit in the name of the trust. Thus, because ABC Trust is being sued, it is appropriate that Trustee's residence, rather than the beneficiaries' residences, be treated as the appropriate domicile for purposes of determining diversity jurisdiction. Thus, answer choices A and B are incorrect. Because complete diversity existed among the parties to the controversy and the amount in controversy exceeded $75,000, it was appropriate to file suit in New York federal court. Answer choice D is incorrect because the government-interest-analysis technique controls in selecting which state's laws should apply in deciding a controversy rather than the issue under these facts regarding a determination of which court may hear the controversy.

* Plaintiff timely filed a notice of appeal with the New York county clerk's office, but failed to timely serve the notice of appeal on Defendants. Plaintiff moved the appellate division to exercise its discretion and consider the appeal, despite the untimely service. Is the appellate division barred from considering the appeal?

* No, the appellate division has discretion to determine whether failure to timely serve is fatal to its ability to consider the appeal.
* Rationale: Answer choice B is correct. If an appellant timely files a notice of appeal, but neglects through mistake or excusable neglect to do another required act within the time limit, the appellate division has discretion to grant an extension to cure the omission. Plaintiffs here timely filed the notice of appeal with the New York county clerk's office, thus authorizing the appellate division to determine whether to exercise its discretion. Thus, answer choices A, C, and D are incorrect.

* Defendant was involved in a car accident with Plaintiff. After the accident, Defendant pled guilty to a charge of failure to obey a traffic-control device. Three years later, Plaintiff filed a civil action against Defendant. Thereafter, Defendant moved to have the judgment of his conviction vacated, and he requested the removal of his previously entered guilty plea based on the fact that he had not been represented by counsel when he pled guilty. The criminal court granted his motion. During the civil action, Plaintiff's attorney questioned Defendant about his prior guilty plea, and defense counsel objected. Is questioning concerning a withdrawn guilty plea permissible?


* No, because the withdrawn guilty plea is not relevant to the civil proceeding.
* No, because questioning regarding the withdrawn guilty plea would violate Defendant's due-process rights.
* Yes, guilty pleas to traffic violations are admissible proof of negligence.
* Yes, but only for impeachment purposes.

Answer choice C is correct. Criminal defendants' withdrawn guilty pleas cannot be used against them as admissions to those crimes for which they are being tried. However, a guilty plea to a traffic violation is not a crime with the same due-process protections, and it is admissible as proof of negligence in a subsequent civil action, provided a defendant is given an opportunity to explain the plea so that the jury might decide whether to afford it any weight. Thus, both answer choices A and D are incorrect. The decision to allow a defendant to withdraw his guilty plea to a traffic infraction because he lacked the benefit of counsel is discretionary with the judge, but it is not permitted as a matter of right because there is not entitlement to the assignment of counsel in a case for a traffic infraction. Because vacating Defendant's plea was not based upon any violation of due process, Plaintiff here is not precluded from asking the Defendant about the guilty plea, and Defendant must be permitted a full and fair opportunity to offer the jury his reasons for the withdrawn plea. Thus, answer choice B is incorrect.

* Plaintiff was injured when she fell while descending a stairway used as a means of access to and egress from the New York City subway. Plaintiff alleges that her foot caught on a metal strip protruding from one of the steps, causing her to fall. The New York City subway is on land that is owned by New York City, but the subway is managed and run by the New York City Transit Authority. Plaintiff brought suit against the New York City Transit Authority asserting that failure to keep and maintain the stairway in a proper and safe condition caused her injury. The New York City Transit Authority sought to have the action dismissed. How should a court decide?

Answer choice C is correct. When a stairwell or approach is primarily used as a means of access to and egress from the common carrier, that carrier has a duty to exercise reasonable care to see that such means of approach remain in a safe condition or, when appropriate, to take such precautions or give such warnings as would protect those using such area against unforeseen danger. Answer choice D is incorrect because when that duty arises, as long as the use is primarily for ingress and egress, it need not be used in this manner exclusively. Courts have long recognized that the duty of care imposed on a common carrier with respect to its passengers requires not only that it keep the transportation vehicle safe, but also that it maintain a safe means of ingress and egress for the use of its passengers. Answer choice B is incorrect because this duty has been applied to those areas owned and maintained by others if "constantly and notoriously" used by passengers as means of approach. Answer choice A is incorrect because this is not a strict liability tort.

* Plaintiff was injured when she fell while descending a stairway used as a means of access to and egress from the New York City subway. Plaintiff alleges that her foot caught on a metal strip protruding from one of the steps, causing her to fall. The New York City subway is on land that is owned by New York City, but the subway is managed and run by the New York City Transit Authority. Plaintiff brought suit against the New York City Transit Authority asserting that failure to keep and maintain the stairway in a proper and safe condition caused her injury. The New York City Transit Authority sought to have the action dismissed. How should a court decide?


* For Plaintiff, because, as a common carrier, the New York City Transit Authority was strictly liable
* For Defendant, because a common carrier has no duty to maintain the stairway on property otherwise owned by a distinct landlord
* For Plaintiff, upon showing Defendant, as a common carrier, failed to exercise reasonable care and to take precautions as would protect those using such area against unforeseen danger
* For Defendant, unless Plaintiff demonstrates the stairway was used exclusively for access to the subway

Answer choice C is correct. When a stairwell or approach is primarily used as a means of access to and egress from the common carrier, that carrier has a duty to exercise reasonable care to see that such means of approach remain in a safe condition or, when appropriate, to take such precautions or give such warnings as would protect those using such area against unforeseen danger. Answer choice D is incorrect because when that duty arises, as long as the use is primarily for ingress and egress, it need not be used in this manner exclusively. Courts have long recognized that the duty of care imposed on a common carrier with respect to its passengers requires not only that it keep the transportation vehicle safe, but also that it maintain a safe means of ingress and egress for the use of its passengers. Answer choice B is incorrect because this duty has been applied to those areas owned and maintained by others if "constantly and notoriously" used by passengers as means of approach. Answer choice A is incorrect because this is not a strict liability tort.

* Testator drafted a will. The will was properly executed, and it distributed most of Testator's estate to Children and Charity. In addition, Testator created a trust for Grandchild separate from the will. The will included a clause that recited, "I give $100,000 to Trustee of my inter vivos trust, to hold and distribute for Grandchild's college education," and instructed that Testator's stock in IBM be distributed in accordance with instructions he had left in his safe-deposit box. In the safe-deposit box was a letter on Testator's letterhead instructing that his IBM shares be liquidated and the proceeds delivered to Testator's mistress. The letter was signed by Decedent and notarized. Are the trust to Grandchild and the letter enforceable?

Answer choice B is correct: the trust but not the letter. New York prohibits the incorporation by reference of other documents into a will, except in the cases of “pour-over” trusts; these are gifts transferred to a trust that was created by a decedent while he was alive. The trust created for the benefit of Grandchild is an inter vivos trust, thereby falling within the exception to the rule. However, the letter is not executed with testamentary formalities and will have no legal effect.

* Petitioner Town hired Respondent, an African-American male, as a sign inspector. One year later, Respondent was suspended pending an investigation into charges of incompetence and misconduct. At a civil service hearing, Respondent claimed that the charges against him were racially motivated, but the hearing officer concluded that Respondent was guilty of several of the charges, recommended a lenient penalty, and found no evidence of racial discrimination. In turn, Town reviewed the findings and decided to fire Respondent. Respondent then commenced an Article 78 proceeding, to overturn the decision to fire him. The firing decision was upheld by the appellate division. Subsequently, Respondent filed a complaint with the New York State Division of Human Rights, charging racial discrimination. Town commenced an Article 78 proceeding, by writ of prohibition, seeking an order preventing adjudication of Respondent's complaint before the State Division of Human Rights. How should the court rule?


* Town's writ of prohibition should be granted because Respondent is collaterally estopped from raising racial discrimination at a human rights hearing after having presented his claim in a civil service proceeding.
* Town's writ of prohibition should be denied as inapplicable because the Division of Human Rights is acting within the scope of jurisdiction.
* Town's writ of prohibition should be denied because a writ is available only in criminal matters.
* Town's writ of prohibition should be granted because pursuant to res judicata, the matter has been adjudged by a court of equally competent jurisdiction.

Answer choice B is correct. Writs of prohibition are available to address whether a body or officer proceeded without or in excess of jurisdiction (CPLR 7803[2]). In this instance, the Division of Human Rights is an appropriate forum to raise a racial discrimination claim. Answer choices A and D are incorrect because collateral estoppel and res judicata are not cognizable arguments under Article 78, which deals with a body acting beyond the scope of its jurisdiction. Answer choice C is a misstatement of law - writs of prohibition may be brought in civil or criminal matters.

* Plaintiff and Defendant contracted for the construction of a parking garage. The contract included a liquidated-damages clause that recited: "Because actual damages for delay in completing the work are impossible to determine, the parties shall be liable for the sum of $1,000 as fixed, agreed, and liquidated damages for each day of delay from the completion date until such work is completed and accepted." While the project was underway, a dispute arose, and Defendant discontinued work on the project before the designated completion date. Plaintiff was forced to retain another contractor to finish the project and, as a result, the garage was not ready to open at the anticipated time. Plaintiff filed suit and sought an award of both actual damages and liquidated damages. Assuming Plaintiff prevails, what measure of damages should the court award?

Answer choice B is correct. The liquidated-damages clause on which plaintiff relies, by its terms, represents an attempt by the parties to anticipate and provide for the specific possibility that Defendant's satisfactory completion of the project might be delayed beyond the agreed-upon date. The clause does not contain clear and unambiguous language, as it should, indicating that it was also intended to apply to Defendant's outright abandonment of the project, an entirely separate eventuality. Therefore, the only reasonable interpretation that can be given to the liquidated-damages provision is that liability for the stipulated sum does not accrue until the contractor has fulfilled its side of the agreement and, consequently, the provision is not available in a case involving a complete renunciation of the contract. Thus, answer choices A, C, and D are incorrect because the contract contained no provision expressly addressing renunciation.

* For the first three months of Baby's life, Grandfather visited with the infant several times per week. Thereafter, Grandfather's relationship with Baby's parents deteriorated, and the parents refused to allow Grandfather to continue to visit with Baby. Grandfather instituted a visitation proceeding in family court, and the parents entered their objections to the request. After lengthy hearings, it was determined that it was in Baby's best interests to have visitation with Grandfather for six hours on the second Sunday of every month. Was the family court correct to grant Grandfather visitation over the parents' objection?

* No, absent a finding that circumstances existed to confer grandparent standing to bring such action.
* Rationale: Answer choice D is correct. New York recognizes some instances in which grandparents have standing to seek custody or visitation. Under certain circumstances, grandparents have standing to seek visitation rights even over the objections of a natural parent. However, the circumstances necessary to confer standing must rise to a level of more than mere animosity between the objecting parties. Therefore, answer choice B is incorrect. By statute, standing for grandparents is automatic only when a parent has died, and otherwise it may be given when equity would see fit to intervene based on the existing circumstances. Thus, answer choice C is incorrect. A court will only apply the best-interests-of-the-child standard if standing has been established. If there is no standing, then the petition will be dismissed. Thus, answer choice A is incorrect.

* In New York, if Widow provides her daughter with a power of attorney to sell her home, does the power of attorney remain in effect after Widow's death or incompetency?

* No, because either the death of a principal or a court order stating that a principal is incompetent automatically terminates an agency.
* Rationale: Answer choice A is correct. In New York, the death of a principal terminates the agency relationship, whether or not the surviving party had knowledge of such death, and once a court adjudicates a principal incompetent, the agency is automatically terminated. Any further action undertaken by the agent is invalid. Therefore, answer choices B, C, and D are incorrect. If Widow, in advance of incapacity, gave her agent daughter a durable power of attorney in writing, expressing the daughter's appointment and Widow's intention that the power not be affected by Widow's future disability or incapacity, Widow may override the loss-of-capacity rule. A principal may revoke a durable power of attorney so long as the principal has capacity.

* Plaintiff sued Defendant Construction Company, alleging Plaintiff was seriously injured while delivering building materials to Defendant Construction Company. Specifically, Plaintiff testified that Defendant Construction Company had engaged him to pick up boxes from a hardware store and deliver them to the job site. At the job site, a roll of roofing material fell from the roof and hit him on the head. Defendant Construction Company testified that its crew stopped work at the site three days earlier, and he claimed he did not engage Plaintiff to deliver any materials on that particular day. After the foregoing evidence was presented, Plaintiff moved for a directed verdict. How should a Court decide?


* For Plaintiff, based on the doctrine of res ipsa loquitor
* For Plaintiff, because irrespective of any outstanding issued of fact, Defendant is liable as a matter of law
* For Defendant, because there remain outstanding issues of fact
* For Defendant, because he owned no duty to Plaintiff

Answer choice C is correct. In order to direct a verdict based on the doctrine of res ipsa loquitor, (1) the event must be of a kind which ordinarily does not occur in the absence of someone's negligence; (2) it must be caused by an agency or instrumentality within the exclusive control of the defendant; and (3) it must not have been due to any voluntary action or contribution on the part of the plaintiff. The summary judgment issue may also be properly approached by simply evaluating the circumstantial evidence. If that evidence presents a question of fact as to the defendant's liability, the case should go to the jury. If the circumstantial evidence does not reach that level and presents a question of fact, the defendant will prevail on the law. Here, there are material questions of fact for the jury as to whether Defendant owed Plaintiff a duty, or whether the event occurred all. Thus, answer choices A, B and D are incorrect.

* Lawyer lectured a group of professional women on their legal rights relating to sexual harassment in the work place. Following a question and answer period in which Attendee shared experiences she had while working at Law Firm, Lawyer approached Attendee and asked for her contact information. Thereafter, Lawyer telephoned and e-mailed Attendee a note expressing outrage over the conduct she described and offered to represent her for free. Attendee reported Lawyer to the Bar Association's disciplinary committee for improper solicitation. Did Lawyer improperly solicit Attendee?

Answer choice C is correct. A prohibited solicitation is an advertisement initiated by or on behalf of a lawyer that is directed to a recipient or group of recipients and a motive for which is pecuniary gain. A state may enact reasonable restrictions with respect to the time, place, and manner of solicitation by members of its Bar. Thus, Lawyer’s actions are not protected by the First Amendment and answer choice D is incorrect. A state’s special interest in regulating members of a profession it licenses justifies the application of rules to regulate solicitation that is misleading, overbearing, or involves other features of deception or improper influence. However, answer choice B is incorrect because not all solicitations carry the potential for such dangers. A state is only justified in prohibiting or limiting solicitations for pecuniary gain under circumstances likely to result in these evils. However, in this instance, Lawyer offered his services to Attendee for free and without the possibility of financial or pecuniary gain, thus, no solicitation was present and answer choice A is incorrect.

* Defendant's photo was taken in connection with a burglary charge. Subsequent to the photo being taken, the burglary charge was dismissed and the file ordered sealed. Despite the file being sealed, the photograph was never returned to Defendant. Thereafter, Defendant was arrested for an unrelated assault, and the photograph from Defendant's sealed file was used to identify him in a photo identification. Defendant moves to suppress the identification on the grounds that use of a photograph from a sealed file violates his Sixth Amendment rights. How should a court rule?


* Grant the motion, because use of a file photograph in a photo array in violation of a court order requiring that the file remain sealed is per se unconstitutional.
* Grant the motion, because use of a photograph from a court ordered sealed file is unduly prejudicial and thus violates due process.
* Deny the motion, provided defense counsel was present to oversee that the identification procedures were not unduly suggestive.
* Deny the motion, because any constitutional violation in using a photograph from a sealed file did not infringe upon a right of Defendant sufficiently to invoke the exclusionary rule.

Answer choice D is correct. The photograph used in the photo array in this case was obtained from a criminal file that had been ordered sealed. Thus, Defendant was entitled to have the photograph returned to him; its retention by the police and use in the photo array violated New York law. However, a statutory violation such as this does not warrant suppression of the identification testimony. There is no suggestion that the identification procedure was conducted under impermissible circumstances, only that the use of the challenged photograph was improper and requires suppression. Absent a challenge of the identification process, the violation of the statute does not in any way affect the proceedings to determine Defendant's guilt or innocence of the crimes charged. A defendant has no inherent or constitutional right to the return of photographs, fingerprints, or other indicia of arrest when charges are dismissed. Thus, answer choices A and B are incorrect. Further, no right to counsel was implicated, as charges in this instance had not yet been filed. Thus, answer choice C is incorrect. Inasmuch as there was no suggestion under the facts that the identification process was inconsistent with any constitutional rights of the defendant, this technical violation does not warrant suppression of the identification testimony.

* In the complaint, Plaintiff alleges that a trip and fall occurred on March 9, 2003, at or near Main Street in Smalltown. Plaintiff alleged that she was crossing a street when she fell on an uneven broken section of the roadway. At her deposition, Plaintiff testified the trip and fall occurred when crossing at an entirely different location, Oak Street. This deposition testimony was corrected by Plaintiff in an "errata sheet" provided on August 9, 2004, four months after Defendant sent Plaintiff a copy of the transcribed deposition on April 3, 2004. In the errata sheet, Plaintiff stated the trip and fall occurred while crossing Main Street, the same location she claimed on her summons and complaint. Plaintiff included an explanation for the misstatement; it was due to confusion. Thirty-one days later, Defendant made a motion seeking an order striking the correction to Plaintiff's deposition testimony as untimely. The discrepancy is significant because Defendant only performed work on Main Street, never on Oak Street. How should the court rule?


* For Defendant because the correction was not made within 30 days.
* For Defendant because the correction was not made within 60 days.
* For Plaintiff because, although untimely, the delay was justified for good cause.
* For Plaintiff because Defendant's motion was untimely.

Answer choice B is correct. A witness may make substantive changes to a deposition transcript, but all changes must be made within 60 days. Thus, answer choice A is incorrect. There is no indication in the facts that the delay in the correction was for good cause. Thus, answer choice C is incorrect. Although laches is an available defense, the CPLR makes no provision for how much time a defendant has to raise an objection to changes submitted, and 31 days is not an unreasonable amount of time. Thus, answer choice D is incorrect.

* Police officers were on routine patrol when they received a radio report that a woman, Complainant, had been assaulted with a gun by her boyfriend. The police picked up Complainant and drove around with her for four or five minutes searching for the suspect-boyfriend. While driving, Complainant saw Defendant seated behind the wheel of a double-parked car. She told the officers that Defendant was a good friend of her boyfriend's and would know where the boyfriend might be. The officers approached the vehicle, and it began moving. The officers followed the vehicle and pulled it over using their lights and a siren. The officers approached the vehicle and observed at the driver's feet a plastic bag containing what appeared to be marijuana. Defendant was arrested and charged with criminal possession of marijuana. Defendant moves to suppress the marijuana evidence on the grounds that the search of the car and subsequent seizure of the marijuana violated his constitutional rights. How should a court rule?


:

1. Deny the motion because the officers acted reasonably in stopping the car, and the marijuana was observed in plain view.
2. Deny the motion because the automobile stop was a non-arbitrary informational stop.
3. Grant the motion because the police had no reasonable suspicion of criminal activity.
4. Grant the motion because the police had no probable cause to search the vehicle.

Answer choice C is correct. The stop of an automobile is a seizure within the meaning of the Fourth Amendment, even if the purpose of the stop is limited and the resulting detention brief. Police stops in New York are legal only pursuant to routine traffic checks to enforce traffic regulations, or when there exists at least a reasonable suspicion that the driver or occupants of the vehicle have committed, are committing, or are about to commit a crime. A stop for any other purpose--including a stop to request information from a driver concerning the whereabouts of a criminal suspect--violates the Fourth Amendment, and thus, answer choices A and B are incorrect. Answer choice D is incorrect because the stop of the vehicle is at issue, and that constitutes a seizure. Absent a legitimate stop, there is no issue as to whether the ensuing search was supported by probable cause.

* Fifty years ago, the owner of a large tract of land in Town began conveying parcels with restrictive covenants to limit development on this land to single-family homes. Plaintiff and Defendant are the current owners of several of the lots that are subject to covenants in a deed within the chain of title, duly recorded, prohibiting "any building except single-family homes." Defendant leased his lot to a cell phone company, which received a permit from Town to construct a cell phone tower on one of the lots. Plaintiff sought an injunction based on the restrictive covenants. How should the court rule?

Answer choice A is correct. Restrictive covenants will be enforced when the intention of the parties is clear and the limitation is reasonable and not offensive to public policy. Here, the intention of the restrictive covenants was clearly to preserve the residential character of the neighborhood by limiting the area to residential use, which limitation is reasonable and does not offend public policy. Upholding Plaintiff's contractual rights in no way denies wireless services, irrespective of whether the site at issue has the best chance of being the only site necessary to meet the needs and demands for wireless telecommunication services for Town, and thus answer choice C is incorrect. Additionally, Town's authority as a governmental entity to issue permits is separate and distinct from Plaintiff's right to enforce the restrictive covenants, a right that only Plaintiff can enforce and, as such, answer choice B is incorrect. Finally, answer choice D is incorrect because an injunction is the appropriate remedy for the enforcement of a covenant.

* In 1990, the certificate of incorporation of Corporation was amended to provide that unanimous consent of the holders of all outstanding shares of the corporation was necessary for the transaction of any business, including amendment to the certificate of incorporation. Petitioner owns 70 percent of the outstanding shares of the corporation and Respondent owns 30 percent. A dispute arose between Petitioner and Respondent and, at a shareholders' meeting, Petitioner voted his 70 percent of shares in favor of a resolution to strike the unanimous consent requirement, and Respondent voted his 30 percent against the resolution. Respondent, as a director of the corporation, refused to sign a certificate of amendment reflecting the deletion of the unanimity provision. Petitioner commenced this proceeding and moved for judgment declaring the resolution valid and enforceable and compelling Respondent to sign the certificate of amendment. How should the court rule?


* For Respondent because amendment of the unanimous consent requirement requires unanimous consent of the shareholders.
* For Petitioner because amendment of the unanimous consent requirement requires two-thirds of the shareholder vote.
* For Petitioner because amendment of the unanimous consent requirement requires a majority of the shareholder vote.
* For Respondent because amendment of the unanimous consent requirement requires unanimous consent of the shareholders and board approval.

Answer choice A is correct. Supermajority provisions for shareholders in a certificate of incorporation may be amended by a two-thirds vote unless the certificate specifically provides otherwise. Here, the certificate of incorporation is sufficiently specific, and the amendment was invalid because it lacked unanimous shareholder approval. Thus, answer choice B is incorrect. Answer choice C is incorrect because an amendment that changes a supermajority requirement for directors' votes requires only a majority when the corporation was formed on or after February 22, 1998. However, when such requirement pertains to shareholder votes, the requirement is for two-thirds irrespective of whether the corporation was formed before or after February 22, 1998. Amendments that change the voting requirement of shareholders do not require board approval; thus, answer choice D is incorrect.

Plaintiff is a Delaware corporation with its principal place of business in New York and an office in Michigan. Plaintiff commenced a lawsuit against Defendant, its insurance broker, for breach of contract and breach of fiduciary duties. Defendant is incorporated in New Jersey and has offices throughout the United States, including in Michigan. The suit was brought in federal court in the Southern District of New York based on diversity grounds. The dispute arises from Plaintiff's claim that Defendant failed to provide adequate notice to Plaintiff's New York office, as required under the terms of the contract, that its insurance policy was set to expire. The insurance policy had been negotiated and entered into in Michigan and had provided coverage to Plaintiff's officers and directors throughout the country. Plaintiff seeks to apply Michigan law, which holds insurance brokers to have a fiduciary duty to clients. Defendant seeks the application of New York law, which does not recognize any special relationship between a broker and its insured. Which law should the court apply?



Answer Choices: 1. New York law.
2. Michigan law.
3. Michigan law for the breach-of-contract claim, and New York law for the breach-of-fiduciary-duty claim.
4. Michigan law for the breach-of-fiduciary-duty claim, and New York law for the breach-of-contract claim.

Answer choice C is correct. In conflict-of-law analyses concerning breach-of-contract claims in insurance litigation, New York courts primarily consider the following facts: (i) location of the insured risk, (ii) the insured's principal place of business, (iii) location of the broker or agent placing the policy, (iv) where the policy was issued and delivered, and (v) where the premiums were paid. When the insured's interests include a wide geographical range, New York courts have applied the law of the state where the policies were executed, issued, and brokered, and where the insured had its principal place of business. Here, because the insurance policy covered individuals throughout the country, the contract law of Michigan, where the policy was issued, executed, and brokered, will be applied. Thus, answer choices A and D are incorrect. However, with respect to the breach-of-fiduciary-duty claim--a tort--the relevant analytical approach is the governmental-interest-analysis technique, which requires application of the law of the jurisdiction having the greatest interest in the litigation. This jurisdiction is almost exclusively the parties' domicile and the locus of the tort. Here, the laws of Michigan and New York, as well as Delaware and New Jersey, arguably have application. However, because the law in question is a conduct-regulating tort, the site of the tort is controlling. In this case, the relevant law involves the failure to provide notice in New York. Thus, New York substantive law applies to the breach-of-fiduciary-duty claim, making answer choice B incorrect.

* In related actions to recover damages for medical malpractice, joined for trial, Defendant 1 brought a motion for summary judgment on her action that was granted in her favor. Thereafter, Defendant 2 brought a similar motion for summary judgment on his action on the same grounds, but 140 days after a note of issue was filed. Plaintiff opposed the second motion for summary judgment, not on the merits but as being untimely. How should the court decide?

* Deny motion for summary judgment because it was untimely and the matters were not consolidated.
* Rationale: Answer choice C is correct. A summary judgment motion made more than 120 days after note of issue is filed is untimely. Thus, answer choices B and D are incorrect. Joining issues for trial, without consolidating an action, is insufficient to make the matters a single cause of action, so they remain separate actions. Accordingly, a timely motion for summary judgment brought by Defendant 1 cannot provide good cause for the untimely filing of a motion by Defendant 2, irrespective of whether the grounds are identical. Thus, answer choice A is incorrect.

* Manufacturer entered into a contract with Plaintiff for the sale of stone panels to build a "pre-engineered" building. The stone panels carried a five-year manufacturer's warranty guaranteeing performance. One year later, shortly after completion, the stone panels began to crack. Four years after entering into the contract, Plaintiff commenced this action. Manufacturer moved to dismiss the complaint, claiming the action was time-barred. How should the court rule?


* The action is time-barred by a four-year statute of limitations that began to run from the date of delivery.
* The action is not time-barred because the four-year statute of limitations did not begin to run until the harm occurred.
* The action is not time-barred because it is subject to a five-year statute of limitations.
* The action is not time-barred because it is subject to a six-year statute of limitations.

Answer choice C is correct. Although a breach-of-warranty action must be commenced within four years after the cause of action accrues, the breach usually accrues upon delivery of the goods, and thus answer choices B and D are incorrect. An exception is made if a warranty explicitly extends the four-year period to the future performance of the goods, and discovery of the breach must await the time of such performance. In this case, the contract expressly recited a five-year warranty, and thus the four-year period is extended to the future performance of the goods. In the event of a warranty of future performance, the cause of action accrues when the breach is or should have been discovered, and thus answer choice A is incorrect.

* Wife and Husband were married 28 years before Wife brought an action seeking a legal separation from Husband. The action resulted in Wife being granted exclusive use and possession of the marital residence, temporary maintenance, and child support. After the action was commenced, the parties continued to live together and filed joint tax returns. More than one year after the commencement of the separation action, Wife won $5,000,000 dollars in a lottery. Immediately thereafter, she filed an action for divorce. Defendant Husband sought an order declaring that the lottery winnings acquired after the commencement of the separation action were marital property. Wife cross-moved to set the date of the commencement of the separation action as the cutoff, after which property acquired would not be considered marital property. How should the court rule?

Answer choice A is correct. The law is well-settled that a lottery prize won during the marriage is marital property and, accordingly, subject to equitable distribution. The question that remains is the statutory determination on when marital property ceases to accumulate. Under New York law, the marital economic partnership should be considered dissolved when a matrimonial action is commenced seeking a divorce, annulment, or declaration of nullity. The commencement of a divorce action terminates the marital economic partnership and provides an objective date to sever the economic ties of the parties. Therefore, answer choices C and D are incorrect. By contrast, separation actions do not constitute the functional or cognizable equivalent of the dissolution of a marital economic partnership or of a marriage itself. Separation decrees merely provide that the parties can live separately and apart either on a temporary or permanent basis. Although separation decrees are often used as a predicate for a divorce in New York, the law requires a specific filing of a divorce action to legally effectuate the divorce objective and terminate a marriage. Thus, answer choice B is incorrect.

* Wife shot Husband four times--twice in the abdomen, once in the chest, and once in the leg. She left without seeking help or assistance for Husband, and he bled to death. Wife told the police that Husband had punched her during an argument and that she lunged at him with a knife. Husband, she said, wrestled the knife away from her and came back at her with it before she was able to grab a gun lying on the counter and shoot him. She stated that she never intended to kill him. Assuming a jury was persuaded by Wife's account, of what statutory crime of homicide is she guilty?


* Murder in the second degree or depraved indifference murder in the second degree.
* Depraved indifference murder in the second degree or manslaughter in the first degree.
* Manslaughter in the first degree or manslaughter in the second degree.
* Manslaughter in the first degree or murder in the second degree.

Answer choice D is correct. Wife's act of shooting Husband in the leg, chest, and abdomen did not, as a matter of law, constitute depraved-indifference murder pursuant to New York law. Depraved-indifference murder arises when the defendant does not intend to kill or seriously injure but nevertheless abandons a helpless and vulnerable victim when the victim is highly likely to die. Here, Wife acted with either the intent to kill or to seriously injure, and thus the crime is not one of depraved indifference, even though she fled without summoning assistance. Thus, answer choices A and B are incorrect. Manslaughter in the second degree involves conduct that is not intentional but that disregards substantial risk that causes the death of another. Because Wife acted with intent to kill or seriously injure, answer choice C is incorrect. Whether Wife will be convicted of intentional manslaughter or intentional murder will depend on whether a jury determines that Wife intended to injure or kill Husband.

* A business dispute spawned three lawsuits. In the first action, Seller sought to recover the purchase price of goods sold and delivered to Buyer. Buyer counterclaimed for breach of warranty for improper manufacture, and Seller denied the allegation of improper manufacture. Thereafter, Seller initiated a second lawsuit to recover the goods from Processor who counterclaimed for its charges and claimed a right to the goods. Finally, Buyer brought a third lawsuit against both Seller and Processor alleging that the goods were defective due to the actions of Seller or Processor or both. May Buyer obtain a single trial against Seller and Processor by joinder or consolidation to determine whether the goods are defective and which party bears responsibility for said defect?


1. No, neither joinder nor consolidation is available under the facts as stated because the actions are based on separate contracts and between separate parties.
2. Yes, Buyer is entitled to a common trial by joinder in a single action, or by a consolidation of actions.
3. Yes, Buyer is entitled to a common trial by joinder. Consolidation is unavailable because Seller and Processor had separate and independent contracts; therefore, this was not the same transaction or occurrence.
4. Yes, Buyer is entitled to a common trial by consolidation. Joinder is unavailable because the nature of the separate relationships with Seller and Processor means that there is no common question of law or fact to sustain a joinder.

Answer choice B is correct. The plaintiff may join as many claims as he may have against the defendants as long as a single event involved all the parties to be joined (in this case, the sale of the goods) and at least one question of law or fact links the claims (in this case, whether the goods were defective). Thus, answer choices A and D are incorrect. When actions involving a common question of law or fact are pending before a court, the court, on motion, may order the actions to be consolidated. Thus, answer choice C is incorrect.

* Plaintiff brought an action against General Contractor for injuries sustained on the job while employed by Subcontractor. The construction project involved the renovation and restoration of a highway overpass. Upon grabbing onto a safety cable installed by General Contractor, the cable came loose causing Plaintiff to fall and sustain injury. General Contractor was found negligent, in part, by the jury. In response, Subcontractor moved to dismiss the claim for indemnification. How should a court decide?

Answer choice B is correct. New York law obligates a subcontractor to indemnify a general contractor only when it is shown that damages were caused by the subcontractor's own negligence. Thus, answer choice A is incorrect. New York also contemplates partial indemnification and is intended to limit a subcontractor's indemnity obligation solely to a subcontractor's own negligence. Thus, answer choice D is incorrect. Answer choice C is incorrect because there is no language within New York statutes that prevents partial indemnification provisions from being enforced in a case when it is shown that both a general contractor and its subcontractor are joint tortfeasors.

* Plaintiff is the owner of a building and entered into a written commercial lease agreement with Defendant for a 10-year term, which provided that "rent . . . payable in equal monthly installments in advance on the first day of each calendar month, without notice or demand." For the first year, Defendant paid the rent on or before the first day of each month. However, in the second year, Defendant was 10 days late paying rent in the first month and 15 days late in the second month. Plaintiff sued Defendant for violating the terms of the lease. How should the court decide?

Answer choice A is correct. An agreement to pay rent on a certain date is generally a material term of a lease, and therefore answer choices B and C are incorrect. Plaintiff may sue Defendant for failure to pay rent on the first of the month because Defendant violated the lease terms. A written lease agreement serves as a complete and clear agreement between the landlord and tenant. Thus, extrinsic and parol evidence is not admissible, making answer choice D incorrect.

* Defendant was the sole general partner in Partnership, a limited partnership. In January, after suffering heavy personal losses in the stock market, Defendant wrote himself partnership checks amounting to more than $1 million to his personal account. After the limited partners and their insurers exhausted efforts to recoup the funds, Defendant was indicted for one count of grand larceny in the second degree and another count of embezzlement. Defendant moved to have the charges dismissed. How should a court rule?


* Deny the motion as to the larceny and embezzlement charge.
* Deny the motion as to embezzlement, but dismiss the larceny charge.
* Deny the motion as to the larceny charge, but dismiss the embezzlement charge.
* Grant the motion.

Answer choice D is correct. Larceny is committed when one wrongfully takes, obtains, or withholds "property from an owner thereof" with intent to deprive the owner of it or appropriate it for oneself or another. "Owner" is defined in N.Y. Penal Law § 155.00(5) as one "who has a right to possession superior to that of the taker, obtainer or withholder." This broad definition is immediately qualified by the declaration that "[a] joint or common owner of property shall not be deemed to have a right of possession thereto superior to that of any other joint or common owner thereof." Id. Embezzlement under New York law is a form of larceny whereby there is a (i) fraudulent, (ii) conversion, (iii) of the property, (iv) of another, (v) by a person who is in lawful possession of the property. A partner cannot be convicted of larceny for the misappropriation of partnership assets because each partner holds title to an undivided interest in the partnership. Under N.Y. Penal Law § 155 "[a] joint or common owner of property shall not be deemed to have a right of possession superior to that of any other joint or common owner thereof." Thus, because both larceny and embezzlement require Defendant to take the property of another, Defendant cannot be charged with larceny or embezzlement for misappropriating firm assets in which he has an ownership interest. Thus, answer choices A, B, and C are incorrect.

* Plaintiff and Defendant are owners of adjoining residential lots. In January 1996, Plaintiff purchased lot A. In 1999, Defendant purchased lot B. Both lots were unimproved land when the parties acquired them, and both parties built homes on their respective lots. On lot A, Plaintiff also built a small shed. While constructing his home, Plaintiff bulldozed and deposited fill and topsoil on Defendant's side yard, dug a trench, and installed water pipes. In addition, Plaintiff constructed a fence to enclose his dog and regularly mowed, graded, raked, planted, and watered the grassy area. In 2010, Defendant surveyed the land and discovered he had title to that portion of the land being maintained by Plaintiff. Upon learning of this, Plaintiff brought an action to quiet title. How should the court rule?


* For Defendant if Plaintiff knew or was on notice that the subject property belonged to Defendant.
* For Plaintiff because he possessed the disputed parcel of land in an open and notorious manner, hostile to the interests of the title owners, and continuously for more than 10 years without Defendant asserting rights.
* For Defendant if he reasonably believed that the property belonged to him.
* For Plaintiff, as long as he can show a reasonable basis to believe the property belonged to him.

Answer choice D is correct. In New York, adverse possession allows ownership to be granted to one who possesses real property in an open and notorious manner, hostile to the owner's rights (without permission), and continuously for more than 10 years. However, an adverse possessor can acquire title only if he has a reasonable basis for the belief that the property belongs to him (a "claim of right"). Here, although the statement in answer choice B is correct, because New York also requires a claim of right, the answer is incomplete (and answer choice D is correct). Answer choices A and C are incorrect because, to determine adverse possession, the court will not focus on the particular person's belief, but instead upon the evidence that justifies a reasonable basis for that belief.

* In March 2006, Decedent was admitted to City Hospital. Decedent died intestate on April 1, 2006, leaving Infant Son as her sole distributee. In addition, Decedent was survived by her sister and mother. On December 22, 2006, letters of guardianship were issued to Decedent's mother, the infant's grandmother. On April 30, 2008, a wrongful-death action was commenced on behalf of Infant Son alleging that Hospital's negligence caused Decedent's death. Thereafter, defendant Hospital moved to dismiss the claim as time barred. Was the court correct in dismissing the action as time barred?

Answer choice B is correct. This is a wrongful-death case, not a personal injury case, so the applicable statute of limitations is two years from the date of death. Thus, answer choice D is incorrect. Because Infant Son was under the disability of infancy when Decedent died, the statute of limitations is tolled until he reaches the age of majority. Thus, answer choice A is incorrect. Answer choice C is incorrect because under the facts as presented, Infant Son was the only distributee eligible to bring this action.

* Defendant was charged with criminal sale of a controlled substance because of his involvement in a narcotics network. Specifically, following police investigation of a cocaine ring, Defendant was charged and tried as an accomplice to the sale of controlled substances. How should the judge instruct the jury regarding the standard of conduct required to find Defendant guilty?


* To find Defendant guilty, a jury must find that Defendant had the specific intent to sell a controlled substance.
* To find Defendant guilty, a jury must find that Defendant should have known that he was engaging in the sale of a controlled substance.
* To find Defendant guilty, a jury must find that Defendant had knowledge that he was engaging in the sale of a controlled substance.
* To find Defendant guilty, a jury must find that Defendant believed that it was probable he was engaging in the sale of a controlled substance.

Answer choice C is correct. N.Y. Penal Law § 20.00 provides that a person may be held criminally liable as an accomplice when he performs certain acts and does so "with the mental culpability required for the commission of" the substantive crime. Caselaw has interpreted this to require that Defendant have knowledge as to what kind of activity he is participating in, but not necessarily mens rea of specific intent to engage in that activity. Thus, answer choice A is incorrect. Answer choice D is incorrect, as it states the standard that is used for the crime of "facilitator" rather than accomplice. Answer choice B is incorrect because it doesn't articulate a recognized standard of culpability under criminal law.

* Plaintiff was employed by Defendant pursuant to an employment-at-will contract for approximately 13 years before he was discharged in January. During each calendar year, Defendant allegedly orally agreed to compensate Plaintiff for his employment services with an annual bonus equal to 20 percent of the company's annual pretax profits. When Plaintiff discovered that his bonus was not in accordance with the parties' alleged oral understanding, he commenced a suit. Defendant moved to dismiss the complaint pursuant to the Statute of Frauds because annual net profits could not be determined until 60 days into the next calendar year. How should the court rule?

Answer choice A is correct. New York law provides that an agreement that cannot be performed within one year will not be recognized or enforceable if it is not in writing. Courts have long interpreted this provision of the Statute of Frauds to encompass only contracts that by their terms have absolutely no possibility of full performance within one year. As long as the agreement may be fairly and reasonably interpreted such that it may be performed within one year, the Statute of Frauds will not act as a bar however unexpected, unlikely, or even improbable it is that performance will occur during that time frame. An at-will employment relationship may be freely terminated by either party at any time for any reason or for no reason, with limited exceptions not relevant here. Under these principles, an at-will employment relationship may usually be completed within one year and ordinarily would not require performance to extend beyond that time. The fact that the calculation of a bonus for such performance would necessarily occur after the passage of one year does not bring it within the bar of the Statute of Frauds. Thus, answer choices B and C are incorrect. Further, answer choice D is incorrect because under the employment relationship as stated, although the amount of the bonus is indefinite, it is determinable even if such calculation would take more than one year.

* Plaintiff commenced a shareholder derivative action against Business Corp. and its board of directors. The complaint alleged that the board of directors wasted corporate assets by awarding excessive compensation to executives and outside directors, but it made no mention of Plaintiff having made a demand on the board to correct the excessive compensation, or having filed suit to recover such excessive compensation. Plaintiff also did not reference why such demand on the board of directors would have been futile. The court dismissed Plaintiff's complaint for failure to state a cause of action because Plaintiff did not make a demand on the board to correct the wrongdoing prior to bringing suit. Plaintiff appealed the decision, claiming that such a demand would have been futile. How should the appellate court decide?


:

1. Affirm, because without further explanation, Plaintiff was not excused from making a demand on the board with respect to the executive compensation claim.
2. Overturn, because a demand on the board to correct its own wrongdoing would be inferred as futile, and thus it need not be explained.
3. Affirm, because director compensation is determined at the discretion of the board, and shareholders have no standing to object to the same.
4. Overturn, because directors awarding their own compensation is a per se violation of their duty of loyalty.

Answer choice A is correct. The plaintiff in a derivative action must make a written demand upon the board of directors to take action, although demand upon the board is not required when doing so would be futile. Absent the recitation of details from which the futility of a demand can be inferred, a plaintiff's failure to make such demand before filing a shareholder derivative suit causes the complaint to fail. Here, Plaintiff failed to explain the futility of the demand, and therefore, answer choice B is incorrect. New York provides boards of directors the authority to fix director compensation unless the charter or bylaws provide otherwise; thus, answer choice D is incorrect. Answer choice C is incorrect because, although courts will not undertake to review the fairness of director salaries unless wrongdoing and oppression or possible abuse of a fiduciary position are shown, shareholders are not completely without standing to object to the same.

Company performed construction work for Public, and Surety furnished Public with a payment bond pledging payment in the event that Public failed to do so. At some point during the construction project, Public failed to pay Company. In March 2002, Company commenced an action against Public and Surety. Company settled its claims against Surety and entered judgment against Public. In 2003, Company became aware of a settlement agreement between Public and Town resulting from litigation. Company served a restraining notice upon Town, and it directed the City Marshal to levy the funds. Unbeknownst to Company, in June 2002, Public previously granted Surety a security interest in all of the sums due from the litigation with Town, and Surety had perfected its security interest by filing a financing statement. Does Surety or Company have a superior security interest in the settlement funds with Town?


* Surety has a superior interest because it attached and perfected its security interest prior to Company filing judgment.
* Company has a superior interest because its claim arose prior to filing the financing statement.
* Company has a superior interest because money judgments are superior to security interests.
* Surety has a superior interest because perfected security agreements are superior to money judgments.

Answer choice A is correct. Creditors can protect their interests in collateral. Under a notice-filing concept, as a matter of public record, personal property is declared encumbered. Secured creditors may avail themselves of the UCC protections by filing financing statements to warn others, such as potential purchasers, transferees, or other creditors, of encumbrances. For a security interest to be valid and enforceable against the debtor and a third party, the debtor must sign a document describing the collateral, and the security interest must attach and be perfected. Here, Surety properly filed its financing statement prior to Company executing its judgment and levy. If a secured creditor's filing statements predate perfection by judgment creditors of their judgment liens, then the rights of the secured creditor come first. Surety's rights are superior to Company's rights. Thus, answer choices B and C are incorrect. Answer choice D is incorrect because priority is based on the timing of filing or perfection rather than the mechanism by which a creditor's claim is secured.

* Plaintiff and Defendant were married for 10 years before Plaintiff started a divorce proceeding against Defendant on the grounds of cruel and inhuman treatment. After the service of the summons and complaint and submission of Defendant's answer, but prior to the granting of a judgment of divorce, Defendant began to have a sexual relationship with his neighbor. Upon discovering that Defendant was in a relationship with the neighbor, Plaintiff sought leave to amend her complaint to add a cause of action for adultery. Should the court permit the addition of an adultery cause of action?



* No, because the purported act of adultery occurred after service of the summons and complaint.
* No, because the purported act of adultery occurred after Defendant consented to jurisdiction by serving an answer.
* No, because Plaintiff cannot bring a divorce proceeding based on more than one grounds for divorce.
* Yes, because while still married, Defendant engaged in sexual relations outside the marriage.


Answer choice D is correct. Adultery is voluntary sexual intercourse with someone other than one's spouse while married. Here, because Defendant's adultery was committed before the judgment of divorce was entered, it occurred during the marriage and therefore fits within the parameters of the adultery statute. Absent a showing of prejudice to Defendant, Plaintiff is not precluded from amending her complaint to include acts of adultery that occurred after the action commenced. Therefore, answer choices A and B are incorrect. Answer choice C is incorrect because there is nothing in domestic-relations law that precludes an individual from seeking a divorce on multiple grounds.

* Plaintiff is a New York-based linen rental business. Its competitor, Defendant, is a nationwide business that rents similar products to commercial customers. Plaintiff alleges that it had a five-year exclusive service contracts with Customer, and that Defendant wrongfully induced Customer to breach its contract with Plaintiff and enter into a rental agreement with Defendant. Plaintiff, by letter, demanded Defendant to discontinue servicing Customer. Defendant denied any wrongdoing and continued servicing Customer. Can Plaintiff sue Defendant for tortiously interfering with its existing customer contract?


:


1. No, because Defendant has a legitimate interest as a competitor to solicit business and make a profit
2. No, because even if Defendant knew about Plaintiff's contract with Customer, it owed Plaintiff no duty
3. Yes, provided that Plaintiff demonstrates a fiduciary relationship with Defendant
4. Yes, provided Plaintiff demonstrates specific intent to induce nonperformance with an existing contract

Answer choice D is correct. New York law recognizes the tort of interference with both prospective and existing contracts, affording greater protection to an already existing contract. Here, because this is a case of contract interference, Plaintiff must show the existence of a valid contract with a third party, Defendant's knowledge of that contract, Defendant's intentional and improper procuring of a breach, and damages. There is no requirement that Defendant has a fiduciary relationship with Plaintiff or owes Plaintiff any kind of special duty. Thus, answer choices B and C are incorrect. A defendant who is simply Plaintiff's competitor and knowingly solicits its contract customers is not economically justified in procuring the breach of contract. In other words, mere status as Plaintiff's competitor is not a legal or financial stake in the breaching party's business that permits Defendant's inducement of a breach of contract; thus, answer choice A is incorrect.

* LLC is a limited liability company organized under the laws of the state of New York. LLC owns a Manhattan apartment building. Plaintiff owns a 25% interest in LLC. Plaintiff initiated, on behalf of the company, a derivative lawsuit on the ground that those in control of LLC had breached their fiduciary obligations to LLC and had authorized leases that were below market value but that benefited the fiduciaries personally. LLC's articles of organization do not expressly recognize derivative lawsuits as a potential remedy. LLC moved to have the action dismissed on the grounds that derivative lawsuits are unavailable against a limited liability company. How should the court rule?



* Grant the motion and dismiss the action because derivative lawsuits are not recognized under the New York Limited Liability Company Law.
* Deny the motion because derivative lawsuits are available against limited liability companies.
* Grant the motion because LLC's articles of organization do not expressly recognize derivative lawsuits as a potential remedy.
* Deny the motion because limited liability companies are governed by the New York Business Corporations Law, which recognizes derivative lawsuits as an available remedy.

Answer choice B is correct. The New York Limited Liability Company Law governs limited liability companies, and thus answer choice D is incorrect. Although the New York Limited Liability Company Law omits any references to derivative suits, the court of appeals has held that this omission does not imply that such suits are prohibited. Courts have repeatedly recognized derivative suits in the absence of express statutory authorization and, in light of this, there is no reason to conclude that the absence of authorizing language in the New York Limited Liability Company Law or within a company's articles of organization would bar a court from entertaining derivative suits by LLC's members. Thus, answer choices A and C are incorrect.

Debtor entered into an installment contract for the purchase of a vehicle. As part of the transaction, Debtor traded in her old vehicle, which had a negative trade-in value (it was worth less than the amount owed on it). The difference between the vehicle's trade-in value and what she owed was rolled into the financing of the new car, resulting in financing totaling $23,000. The lien against the trade-in was paid off by Dealer, and the security interest in the new vehicle was assigned to Creditor. Nearly two years after purchasing her new vehicle, Debtor filed for bankruptcy. As part of her bankruptcy plan, Debtor proposed that she retain possession of the vehicle and that Creditor's secured claim would be reduced to $10,000, representing the retail value of the vehicle, and the remaining amount owed Creditor would be treated as an unsecured claim. Creditor objected, claiming that the entire debt, including the negative equity from the prior vehicle, was entitled to be treated as secured by the vehicle.

Can negative equity be part of a purchase-money security interest arising from Debtor's purchase of her new car?


A. No, negative equity is not an expense incurred in connection with the acquisition of a new car.B. No, because a purchase-money security interest is limited to the market value of the item financed.C. Yes, because Creditor is entitled to determine the extent of the debt it is willing to secure with collateral.D. Yes, because negative-equity financing can be deemed part of the price of an item financed.

Answer choice D is correct. A security interest in goods is a purchase-money security interest to the extent that the goods are purchase-money collateral to that security interest. Purchase-money collateral refers to goods that secure a purchase-money obligation incurred in that collateral. Price of collateral includes obligations for expenses incurred in connection with acquiring rights in the collateral, sales taxes, duties, finance charges, interest, freight charges, costs of storage in transit, demurrage, administrative charges, expenses of collection and enforcement, attorney's fees, and other similar obligations. The term "price" is broad enough to encompass negative-equity financing as part of what was intended as "other similar obligations." Just as "finance charges" and "interest" constitute obligations that are paid over and above a vehicle or other item's actual cost, negative equity from a prior vehicle is part of the overall price. Such price is not limited to market value. Creditor does not have the authority to determine the extent of the debt it is willing to secure with collateral. Thus, answer choices A, B, and C are incorrect.


Late in the afternoon on a bright, summery June day, a group of young people from Delaware County set out for a local lake to go swimming. Four of those within the group piled into a vehicle operated by 17-year-old Defendant. The vehicle had no mechanical defects and nearly new tires. Defendant was in possession of a junior license which imposed several restrictions. One of the restrictions provided that a vehicle could not be operated with more than two passengers under 21 years of age, and driver was required to ensure all passengers buckled their seat belts. On this trip to the lake, none of the four passengers - a 14-year-old, a 15-year-old, a 17-year-old and an 18-year-old - wore a seat belt. Defendant, however, did. The speed limit on the roadway was 55 mph, but at a steep curve the speed limit was reduced to 40 mph. Although Defendant had consumed no alcohol or drugs prior to driving, he lost control of the vehicle. The car crashed, resulting in the deaths of three of the passengers. An accident reconstructionist determined Defendant's speed to have been between 70 and 75 mph.
Are the facts sufficient to make out an action for criminally negligent homicide?


A. Yes, because excessive speed is sufficient to support a criminally negligent homicide charge.B. No, because excessive speed does not support a criminally negligent homicide charge as a matter of law.C. Yes, because the violations of the junior license were evidence of criminal negligence.D. No, because nothing in the facts suggests Defendant had the requisite mens rea.

Answer choice D is correct. "A person is guilty of criminally negligent homicide when, with criminal negligence, he causes the death of another person." (N.Y. Penal Law § 125.10) A person acts with criminal negligence with respect to a result when he fails to perceive a substantial and unjustifiable risk that such result will occur or that such circumstance exists. The risk must be of such nature and degree that the failure to perceive it constitutes a gross deviation from the standard of care that a reasonable person would observe in the situation. In short, although speeding can be a factor involved in creating culpability - and, thus, answer choice B is incorrect - it takes an additional affirmative act to transform "speeding" alone into a "gross deviation." Thus, answer choice A is also incorrect. There must be conduct by Defendant that exhibits the kind of "seriously blameworthy" carelessness whose "seriousness would be apparent to anyone who shares the community's general sense of right and wrong." Here the behavior is negligent, but is not necessarily blameworthy. A 17-year-old, who badly misgauges his ability to handle road conditions and fails to comply with license restrictions limiting the number of passengers and requiring passengers to wear seatbelts, is not engaging in the kind of seriously condemnatory behavior that the legislature envisioned when it defined "criminal negligence," even though the consequences here were fatal. Thus, answer choice C is also incorrect.

Which of the following civil suits, if any, mandate that the matter be tried before a jury?


A. AnnulmentB. Medical malpractice seeking recovery in excess of $250,000C. DefamationD. None of the above

Answer choice D is correct. Generally, issues of fact are tried by a jury in civil actions involving judgments for money; actions for ejectment, dower, waste, nuisance, and recovery of chattel; or for claims involving real property, or any other action in which a party is constitutionally or statutorily entitled to a trial by jury. However, despite a party having a right to a jury trial under those circumstances, none of those causes of action mandates a trial by a jury; and a party can waive a trial by jury either by not filing a demand for jury trial, or by filing a waiver of such right with the court. Accordingly, answer choices A, B, and C are incorrect.

May a board of directors indemnify a director for legal fees incurred in defending herself in a lawsuit based on actions taken on behalf of the corporation?


A. No, the decision to indemnify an officer or director for legal fees must be made by shareholders.B. Yes, an officer or director has an absolute right to be indemnified for legal fees and expenses incurred as a consequence of being sued for actions taken on behalf of a corporation.C. No, unless the officer or director is absolved of wrongdoing or the corporation has agreed to indemnify her fees for actions taken in good faith.D. Yes, provided indemnification of fees has been approved by the board of directors.

Answer choice C is correct. Indemnification is required when a director is successful in defending against the suit and is permitted when the actions are in good faith and not against the best interests of the corporation. Indemnification may be authorized by the certificate or bylaws to be by resolution of the shareholders, by resolution of the directors, or by agreement. Thus, answer choice A is incorrect. Indemnification is prohibited when directors act in bad faith, are deliberate and dishonest, or personally gain that to which they are not legally entitled. Thus, answer choices B and D are incorrect.

Retired Attorney retired from the practice of law and stopped paying legal dues or participating in continuing legal education requirements. Retired Attorney provided some pro bono legal services and continued to use letterhead representing himself as an attorney.

Were Retired Attorney’s actions of providing pro bono services or use of letterhead identifying him as an attorney violate the Rules?


A. Yes, Retired Attorney is precluded from practicing law or using letterhead identifying himself as an attorney.B. No, Retired Attorney is not precluded from continuing to practice law provided he does not charge a fee and his letterhead indicates he is retired.C. No, Retired Attorney is not precluded from practicing law provided he does not charge a fee and there is no prohibition on the use of letterhead identifying himself as an attorney.D. There are no limitations on Retired Attorney’s ability to continue to practice law provided his letterhead identifies him as retired.

nswer choice C is correct. Attorneys who certify that they are retired from the practice of law are exempt from continuing legal education requirements and biennial dues, but may still practice law without compensation. Thus, answer choices A and D are incorrect. Retired attorneys are not as a matter of law precluded from using professional letterhead. Attorneys are allowed to use professional letterhead if doing so does not violate any statute or court rule, and is not false, deceptive, or misleading. New York allows only those who are licensed and admitted to practice law to hold themselves out as “attorneys-at-law.” There is no requirement that Retired Attorney disclose on professional letterhead that he or she is retired. Thus, answer choice B is incorrect.

By written settlement agreement, Plaintiff and Defendant agreed to drop their pending lawsuits. The pending lawsuits arose from a dispute between the parties in which Plaintiff claimed that Defendant owed it $300,000 after withdrawing from a joint venture to construct a nightclub. All pending claims between the parties were discharged pursuant to the settlement agreement. The settlement agreement, among other things, called for Defendant to provide to Plaintiff a "right of use" to a certain mailing list free of charge for a five-year period. At the time of the settlement, the mailing list in question was in the process of being compiled by Defendant. The agreement stated that "the mailing list shall be compiled and provided pursuant to a definitive agreement to be mutually agreed upon and, in any event, containing terms and conditions consistent with those described herein." Thereafter, Defendant and Plaintiff were unable to mutually agree to terms relating to the mailing list. Plaintiff filed suit, alleging a breach of the settlement agreement.

How should the court rule?


A. For Plaintiff, because failure to provide the mailing list was a breach of contract.B. For Plaintiff, because failure to come to terms relating to the mailing lists was a breach of the settlement agreement.C. For Defendant, because the inability to come to terms relating to the mailing list was a failure of a condition precedent.D. For Defendant, because the term regarding the mailing lists was illusory and rendered the settlement agreement void.

Answer choice C is correct. The parties entered into a valid settlement agreement. Note that courts favor settlements and are reluctant to set them aside. Although there was a valid settlement agreement in this case, the obligations never became enforceable because agreed-upon conditions were not met. Most conditions precedent describe acts or events that must occur before a party is obliged to perform a promise made pursuant to an existing contract. Here, the settlement agreement contemplated a condition precedent: the occurrence of a negotiation and execution of an additional agreement regarding the mailing list. As such, absent a showing of bad faith, it cannot be said that Defendant breached the settlement agreement by failing to provide the mailing list. Thus, answer choices A and B are incorrect. Answer choice D is incorrect because the promise requiring the parties to come to terms regarding the mailing list was not one-sided or lacking mutuality, and thus it does not render the agreements void or unenforceable.

Defendant was president and majority shareholder of a small family business engaged in waste disposal. He was on trial for one count of criminal possession of a forged instrument in the second degree stemming from a multiparty prosecution for commercial hazardous waste disposal crimes. Defendant took the stand in his own defense. Thereafter, in an effort to discredit Defendant, the prosecution, over defense counsel's objection, asked Defendant about prior convictions he had for environmental crimes committed by himself and his corporation. Did the trial court make an error in allowing the prosecution to ask Defendant about prior environmental crimes?


A. Yes, because the environmental crimes involving Defendant's corporation are more prejudicial than probative.B. Yes, because environmental crimes do not pertain to veracity.C. No, because the crimes charged are prior crimes probative of the defendant's propensity to commit crimes.D. No, because the court may admit prior criminal acts for impeachment purposes.

Answer choice D is correct. The exclusion or inclusion of prior convictions for impeachment purposes is a discretionary determination for the court. Because Defendant is the owner and president of a small family business, it is not error to admit certain convictions of the corporation to impeach his credibility. Such evidence can be probative of Defendant's credibility as owner and officer. Thus, answer choice A is incorrect. The use of prior convictions to impeach credibility of a defendant, who was jointly tried with his company, is not erroneous as a matter of law; it can be considered probative of credibility, even when the crime itself is not one involving veracity, and thus answer choice B is incorrect. Prior crimes or prior bad acts are not admissible to prove or bear on a defendant's criminal propensity or responsibility for a crime, and thus answer choice C is incorrect; but such prior crimes or bad acts can be used to affect credibility and defensive explanations or deflections of responsibility.

Attorney sent Adversary an e-mail with a proposed settlement offer. The e-mail indicated in a line denoted “cc:” that the communication was being copied to Attorney’s client. Adversary responded to the e-mail using the “Reply All” function with a counteroffer that sent a response to both Attorney and Attorney’s client.

Did Adversary violate the no-contact rule by replying to both Attorney and Attorney’s client?


A. Yes, because all communications with Attorney’s client, a party known to be represented by counsel, are barred.B. Yes, unless there was prior consent to such communication.C. No, because by including his client in the original e-mail, Attorney waived the no-contact rule.D. No, because communications regarding settlement are excluded from the no-contact rule.

Answer choice B is correct. The no-contact rule prohibits a lawyer from sending a letter or e-mail directly to a represented person without first obtaining consent to the direct communication, unless otherwise authorized by law. Answer choice A is incorrect because it is overbroad, and fails to account for the possibility of such consent. Prior consent to the communication means actual consent and preferably express consent. Answer choice C is incorrect because including his client in the original e-mail does not serve as a waiver of the no-contact rule by Attorney. Answer choice D is incorrect because there is no settlement communication exception to the no-contact rule.

Corporation went out of business and was dissolved for nonpayment of taxes. Two years later, after dissolution, Corporation was sued for negligence. Corporation answered the complaint and brought a third-party action against Company for contribution. Company moved to dismiss the third-party complaint on the ground that Corporation lacked capacity to bring suit. How should the court rule?


A. Grant the motion to dismiss the complaint because a dissolved corporation has no capacity to bring a law suit.B. Grant the motion to dismiss the complaint because filing suit is not a part of winding up a corporation's affairs.C. Deny the motion to dismiss and permit the third-party complaint because dissolution never precludes a corporation's ability to file suit.D. Deny the motion because an action for indemnification or contribution is a part of a corporation's winding up of business affairs.

Answer choice D is correct. The dissolved corporation may continue to act as the corporation for the purpose of winding up, and the corporation may sue and be sued in its corporate name. Accordingly, answer choices A, B, and C are incorrect. Here, Corporation had the capacity to bring its third-party suit, so long as that activity was part of winding up its affairs. To wind up its affairs, Corporation had to either settle or defend the suit brought against it. To assert a third-party claim for contribution or indemnification is a normal part of defending a claim; therefore, answer choice D is correct.


While attending college, Patty worked part-time as an ocean lifeguard for City. During this period, her immediate supervisor repeatedly subjected her to sexual harassment, threatening to fire Patty if she refused to engage in sexual relations with him. Patty filed a lawsuit against City for damages arising from the harassment, based on the claim that her supervisor was an agent of City.

Will Patty prevail in her lawsuit against City?


A. Yes, because employers are subject to liability for employees' torts committed while acting in the scope of employment, under respondeat superior.B. No, because employers are not vicariously liable for the intentional torts of their employees that fall outside the scope of their employment.C. Yes, because an employer can be held vicariously liable for the tortious conduct of a supervisor abusing his supervisory authority.D. No, because such harassment is an incidental hazard known for the job of a lifeguard, and therefore it is not actionable.

Answer choice C is correct. An employer is vicariously liable for the tortious conduct of a supervisor abusing his supervisory authority. Answer choice A is incorrect because sexual harassment is not a tort within the job description of any supervisor or any other worker in any reputable business. Answer choice B is incorrect because employers can be held liable for the intentional torts of employees when an employee's acts are foreseeable and the employer should bear the resulting costs of doing business. Answer choice D is incorrect because sexual harassment is not a hazard incidental to employment.

Plaintiff Partnership was a partnership comprising 10 partners, including Defendant. Partnership signed a release promising that in consideration for Partnership performing its services, Partnership would not sue for negligent or wrongful acts committed by the partnership. Subsequently, Partnership, led in its efforts by Defendant, was negligent in performing its obligations. Partnership sued Defendant individually, and Defendant brought a motion to dismiss, alleging the release of Partnership from any potential liability, and cited the signed release of every individual partner's responsibility by Partnership.
Does the written release of Partnership from liability automatically release Defendant from liability in this action taken by Partnership?


A. Yes, a release of a partnership necessarily releases individual partners from liability.B. No, a release of a partnership does not release individual partners from liability in their individual capacity.C. Whether a release of a partnership from liability necessarily extends to the individual partners is dependent upon the terms of the partnership agreement.D. Although a release of a partnership does not necessarily release an individual partner from liability, such liability is limited to the amount of an individual partner's contribution.

Answer choice B is correct. A general release of a partnership does not, in and of itself, release individual partners in their individual capacities. Each partner is liable for any negligent or wrongful act committed by him or under his direct supervision or control. Thus, answer choice A is incorrect. Answer choice C is incorrect because whether a release from liability is accorded to a partnership or the individual partners will be determined by the language of the release rather than the partnership agreement. Finally, what distinguishes a partnership from a corporation is that absent an agreement to the contrary, a partner is individually liable for the actions of its partners and of the partnership, whereas, in a corporation, the general rule is that a shareholder's liability is limited to the amount of his contribution. Thus, answer choice D is incorrect.

Defendant, a shoe importer/exporter, telephoned Plaintiff, a shoe wholesaler. Defendant and Plaintiff regularly did business, both buying and selling shoes to each other. In December, the parties discussed the purchase of shoes. After the conversation, Plaintiff sent Defendant a dated confirmation slip, which included the following terms: "WE HAVE SOLD TO YOU 250,000 SHOES." The confirmation slip called for delivery the following August. Defendant's treasurer signed the slip and returned it to Plaintiff five days later. By letter dated in July, Plaintiff notified Defendant that he was awaiting instructions on how to deliver the shoes to Plaintiff. Defendant responded by letter stating that Plaintiff was mistaken, and that Defendant intended to sell shoes to Plaintiff rather than make a purchase. To cover, Plaintiff sold the shoes at a loss and brought suit against Defendant. Plaintiff moved for summary judgment, and Defendant sought admission of parol evidence to prove that the parties' intent was to purchase shoes from rather than to sell shoes to Plaintiff.

How should the court rule on Defendant's request for admission of parol evidence?


A. Deny the motion because triable issues of fact exist as to whether the confirmation slip signed by Defendant was intended to be the final expression of Defendant and Plaintiff's agreement.B. Deny the motion because parol evidence does not bar evidence offered to show that a contract between the parties never existed.C. Deny the motion because the confirmation slip failed to include an express indication that Defendant and Plaintiff intended the slip to be the final expression of their agreement.D. Grant the motion because the confirmation slip was intended to be the final expression of Defendant and Plaintiff's agreement.

Answer choice D is correct. When the terms of a contract are in a writing intended by the parties to be a final expression of their agreement, the terms cannot be contradicted by extrinsic evidence of a prior agreement or a contemporaneous oral agreement. However, terms can be explained or supplemented by course of dealing or performance between the parties. Here, the essential terms of the transaction are plainly set forth in the confirmation slip. Defendant's agent signed and returned the confirmation slip signifying acceptance of these terms. Nothing in the confirmation slip suggests it was to be a memorandum of a preliminary or tentative understanding with respect to these terms. On the contrary, the slip clearly demonstrates the final expression of the parties' agreement that "WE HAVE SOLD TO YOU 250,000 SHOES." These terms cannot be contradicted by extrinsic evidence but they may be explained or supplemented by showing course of dealing or performance between the parties. The confirmation does not describe a bargain to be made in the future but expresses a meeting of the minds as to completed essential terms - the sale of 250,000 shoes. As such, answer choice A is incorrect. Answer choice B is incorrect because, although UCC § 2-202 does not bar the use of parol evidence to show that there was never a contract, the only dispute here is whether the agreement was for the purchase or sale of shoes. There is no requirement that the writing itself contain express terms that this is the final expression of the agreement. Thus, answer choice C is incorrect.

Tenant of a retail store granted his landlord a security interest in connection with his lease of the premises, which was properly perfected by filing. A fire consumed the store, destroying the collateral. Insurance Carrier paid the loss proceeds directly to Tenant, who was its policyholder and the only loss payee named in the policy. Landlord sued Insurance Carrier, claiming that Insurance Carrier should have given the loss proceeds to it as a security interest holder.

How should the court rule?


A. For Insurance Carrier because only Tenant was named in the insurance policy.B. For Landlord because he had a properly perfected a security interest.C. For Insurance Carrier because it was not a party to the security agreement between Landlord and Tenant.D. For Landlord because Insurance Carrier was on constructive notice of Landlord's security interest.

Answer choice A is correct. Secured creditors may avail themselves of Article 9 protection by filing financing statements that serve to warn others, such as potential purchasers, transferees, or other creditors, of encumbrances. As such, creditors gain protection, as do others who choose to do business with debtors. The rights of a secured creditor are also protected by according a secured creditor an interest in property that the debtor receives as replacement for the encumbered property. However, as to insurance policyholders, an insurance carrier paying proceeds to the proper recipient under a policy is more like an account debtor who makes payment without actual notice that the amount due has been assigned and that payment is to be made to the assignee. Thus, absent actual notice, answer choice D is incorrect. A secured party always has the option of having itself named in the insurance contract as the loss payee or as an additional insured on the risk. Answer choices B and C are therefore incorrect.

Defendant was suspected of a burglary in Nassau County, New York. He was arrested in Nassau County on an Ohio warrant after a police teletype confirmed that he was wanted for robbery in that state. Defendant had counsel on the pending Ohio charge, but the police made no inquiry about Defendant's representation when they called Ohio to confirm the warrant or when questioning Defendant. After receiving Miranda warnings, Defendant waived his right to counsel on the New York charges and admitted his involvement in the Nassau County burglary. Defendant moved to suppress his confession on the grounds that he could not waive his Miranda rights in the absence of counsel.
How should the court rule?


A. Deny the motion because legal representation on unrelated charges does not trigger the right to counsel in New York.B. Grant the motion because legal representation on any charges triggers the right to counsel in New York.C. Deny the motion because legal representation on out-of-state charges does not trigger the right to counsel on New York charges.D. Grant the motion because legal representation on unrelated charges when law enforcement should have known that Defendant had counsel is sufficient to trigger the right to counsel on New York charges.

Answer choice A is correct. In New York, the right to counsel does not require exclusion of statements made to police in response to inquiries about crimes unrelated to those on which the suspect had representation. Thus, answer choices B and D are incorrect. In this instance, Defendant knowingly and voluntarily waived his Miranda rights, and therefore any statements concerning charges unrelated to the crimes for which Defendant has counsel are admissible. Further, answer choice C is incorrect because there is no logical reason to distinguish the effect of charges that are out of state, as opposed to in state, in evaluating the effect on a defendant's right to counsel.

A dispute arose between Insured and Insurance Company as to whether Insured was entitled to renew his life-insurance policy. Insurance Company contended that an enlarged heart made Defendant ineligible to renew his policy. At trial, Insurance Company was unable to locate a copy of the x-ray purporting to show Insured's heart. Instead of this evidence, Insurance Company sought to submit an x-ray report, recorded by a physician as part of the underwriting process, along with expert testimony concerning the physician's impression of the x-ray, in support of its claim that Defendant had an enlarged heart. Insured objected to the admission of this evidence. How should the court rule?


A. Exclude the evidence because the best-evidence rule bars admission of secondary evidence to prove the contents of a lost x-ray.B. Admit the evidence upon proof of the contents of the lost document by competent secondary evidence pursuant to the best-evidence rule.C. Exclude the evidence because x-rays are photographs and not writings or documents whose contents are subject to the best-evidence rule.D. Admit the secondary evidence, provided it is relevant.

Answer choice B is correct. The best-evidence rule requires the production of an original document when its contents are at issue. Under a long-recognized exception to the best-evidence rule, secondary evidence of the contents of an unproduced original may be admitted if the trial court is satisfied that the proponent has sufficiently explained the original's unavailability and has not caused the document to be unavailable by acting in bad faith. New York courts have applied this standard formulation of the best-evidence rule, and this exception, to unproduced x-rays and the derivative evidence offered in their place to describe them. Thus, both answer choices A and C are incorrect. No categorical limitations are placed on the types of secondary evidence that are admissible. Nonetheless, before the trial court rules on the admissibility of the proffered evidence, the proponent of such derivative proof has the burden of establishing that it is a reliable and accurate portrayal of the original. Consequently, answer choice D is incorrect because the fact that the evidence is relevant is not enough, alone, to ensure its admissibility.

Debtor entered into an installment contract for the purchase of a vehicle. As part of the transaction, Debtor traded in her old vehicle, which had a negative trade-in value (it was worth less than the amount owed on it). The difference between the vehicle's trade-in value and what she owed was rolled into the financing of the new car, resulting in financing totaling $23,000. The lien against the trade-in was paid off by Dealer, and the security interest in the new vehicle was assigned to Creditor. Nearly two years after purchasing her new vehicle, Debtor filed for bankruptcy. As part of her bankruptcy plan, Debtor proposed that she retain possession of the vehicle and that Creditor's secured claim would be reduced to $10,000, representing the retail value of the vehicle, and the remaining amount owed Creditor would be treated as an unsecured claim. Creditor objected, claiming that the entire debt, including the negative equity from the prior vehicle, was entitled to be treated as secured by the vehicle.

Can negative equity be part of a purchase-money security interest arising from Debtor's purchase of her new car?


A. No, negative equity is not an expense incurred in connection with the acquisition of a new car.B. No, because a purchase-money security interest is limited to the market value of the item financed.C. Yes, because Creditor is entitled to determine the extent of the debt it is willing to secure with collateral.D. Yes, because negative-equity financing can be deemed part of the price of an item financed.

Answer choice D is correct. A security interest in goods is a purchase-money security interest to the extent that the goods are purchase-money collateral to that security interest. Purchase-money collateral refers to goods that secure a purchase-money obligation incurred in that collateral. Price of collateral includes obligations for expenses incurred in connection with acquiring rights in the collateral, sales taxes, duties, finance charges, interest, freight charges, costs of storage in transit, demurrage, administrative charges, expenses of collection and enforcement, attorney's fees, and other similar obligations. The term "price" is broad enough to encompass negative-equity financing as part of what was intended as "other similar obligations." Just as "finance charges" and "interest" constitute obligations that are paid over and above a vehicle or other item's actual cost, negative equity from a prior vehicle is part of the overall price. Such price is not limited to market value. Creditor does not have the authority to determine the extent of the debt it is willing to secure with collateral. Thus, answer choices A, B, and C are incorrect.

Charles created an unincorporated charitable association called CARP, which was organized to promote the ethical treatment of marine life. The organization boasted a membership of more than 500 individuals. CARP maintained a bank account, rented office space, and maintained a website through which members could keep up to date on protests, rallies, and volunteer opportunities. Dirk was an active member who frequently reviewed the website and attended rallies. CARP posted on its website an announcement soliciting volunteers to help stop the opening of a restaurant known as the Green Claw, and it instructed members to throw bricks through the restaurant's windows. After reading the post on the website, Dirk took a brick and threw it through the window of the Green Claw. Subsequently, the Green Claw sued CARP and Charles on the ground that Dirk's actions were that of their agent.

Will the Green Claw suits be allowable in court against CARP and Charles?


A. The suit can be maintained against CARP but not Charles.B. The suit can be maintained against Charles but not CARP.C. The suit can be maintained against CARP and Charles.D. The suit cannot be maintained against either CARP or Charles because principals are not liable for the illegal acts of their agents.

Answer choice B is correct. In New York, an unincorporated association lacks the capacity to form agency relationships and cannot be held liable as a principal. However, members can personally be held liable for actions authorized through such associations. Thus, answer choices A and C are incorrect. Because an agency relationship does not exist, the association and Charles are not principals, and Dirk is not an agent. Thus, answer choice D is incorrect.


In a homicide and weapons possession prosecution, the prosecution sought to admit as evidence tapes of 911 telephone calls made by an anonymous caller at 7:11 on the morning of the murder. The recording reported: "There was a murder over here at about 4:30 or 5:00 o'clock. The police are downstairs, and the guy that did it is standing outside.* * * Hello. They're grabbing the wrong guy. * * * The guy they're looking for is going into the house. The number of the house is 24. * * * They got the wrong guy right now. The guy that they have right now has yellow and brown striped shorts and the guy they're supposed to be getting has on plain brown." Is the 911 recording admissible?


A. The recording is admissible as relevant identification evidence.B. The recording is inadmissible hearsay.C. The recording is admissible as a present sense impression.D. The recording is inadmissible unless the anonymous caller is definitively identified.

Answer choice B is correct. Although the evidence may be relevant to identify the defendant, the call amounts to an out-of-court statement that is offered to prove the truth of the matter asserted. Thus, answer choice A is incorrect. Answer choice C is incorrect because the declarant on the tapes was referring to an observation made at the time of the murder several hours earlier. It is the passage of time rather than the anonymity of the caller that renders the statement inadmissible, and thus answer choice D is incorrect.

Lawyer was retained by Client to represent him in a criminal matter. Lawyer successfully negotiated a plea bargain in which the court agreed to a conditional discharge of a misdemeanor. The conditional discharge was conditioned on Client not being arrested for a year. A week before the court entered the plea, Client informed Lawyer he had been arrested.

Must Lawyer disclose the arrest to the prosecutor or the court?


A. Yes, because allowing the court to believe Client had not been arrested would constitute a fraud on the tribunal.B. Yes, but only if client is unable or refuses to advise the court or prosecution himself.C. No, because the communication was told to Lawyer in confidence.D. No, because neither Client nor Lawyer have any affirmative obligation to come forward with the information advising the court or prosecution of the arrest.

Answer choice A is correct. A lawyer who represents a client before a tribunal and knows that the client intends to engage, is engaging, or has engaged in criminal or fraudulent conduct related to the proceeding must take reasonable remedial measures, including disclosing such facts to the tribunal. Representing or allowing the court to believe that Client had not been arrested after an arrest took place would constitute a fraud on the tribunal. Thus, answer choice D is incorrect. There is no confidences and secrets exception to the rule. Thus, answer choice C is incorrect. In addition, prior to April 1, 2009, the Rules required an attorney to “call upon the client to rectify the same,” before obligating attorney to make a disclosure; that requirement has been dispensed with under the new rules. Thus, answer choice B is incorrect.

During arrest processing, law-enforcement officials, over Defendant's objection and without providing Miranda warnings or having defense counsel present, forced Defendant to remove clothing and have photographed his various tattoos. Defendant's tattoos included the phrase "All Cops are Bastards," a Nazi swastika, and a depiction of a skinhead. Defendant moved to have the photographs excluded from evidence as violating his Fourth, Fifth and Sixth Amendment rights.
Which of the following is a correct statement?


A. The photographs violated Defendant's Fourth Amendment expectation of privacy during his arrest processing.B. The photographs violated Defendant's Fifth Amendment right against self-incrimination because forcing Defendant to remove his clothing compelled Defendant to reveal thoughts and beliefs without the benefit of Miranda warnings.C. The photographs did not violate Defendant's Fifth Amendment right against self-incrimination because the photographs depict physical characteristics.D. The photographs violated Defendant's Sixth Amendment right to have counsel present during a critical stage of proceedings against Defendant.

Answer choice C is correct. Defendants do not have a legitimate Fourth Amendment expectation of privacy during the arrest process, and thus answer choice A is incorrect. The Fifth Amendment privilege against self-incrimination protects an accused only from being compelled to testify against himself or otherwise provide evidence of a testimonial or communicative nature. Tattoos are not testimonial, and thus answer choice B is incorrect. Answer choice D is incorrect because processing a defendant after arrest is not a significant judicial activity that would implicate the Sixth Amendment right to counsel.

Plaintiff and Defendant entered into a three-year contract under which Defendant would provide services modifying and adapting software developed to meet the requirements of municipal agencies to which Plaintiff hoped to sell the software. The contract contained a broad limitation-of-liability clause, which promised that "in no event shall Plaintiff or Defendant be liable for consequential damages, even if such damages were foreseeable." The contract went on to say that the limitation-of-liability clause did not extend to "consequential damages resulting from willful acts or gross negligence." One year into the contract, Defendant requested an upward adjustment in compensation, and when Plaintiff denied the request, Defendant refused to continue working. Plaintiff commenced this action and sought expectation damages and consequential damages for Defendant's willful violation.

How should the court rule?


A. Plaintiff can recover expectation and consequential damages.B. Plaintiff can recover expectation damages only.C. Plaintiff can recover consequential damages only.D. Plaintiff can recover neither expectation nor consequential damages.

Answer choice B is correct. Generally, expectation damages are available in any breach, but consequential damages are recoverable only if they were reasonably foreseeable. Answer choice C is incorrect because one can never recover consequential damages and not expectation damages. Answer choice D is incorrect because there was clearly a breach, and the contract says nothing about barring expectation damages. A limitation-on-liability provision in a contract represents the parties' agreement on the allocation of the risk of economic loss in the event that the contemplated transaction is not fully executed, and courts will typically find them enforceable. The issue here is what the parties intended by "willful acts." Generally, in the law of contract damages (as opposed to tort damages), whether the breaching party deliberately rather than inadvertently failed to perform contractual obligations should not affect the measure of damages - an intention not to perform a contract does not bring on heavier damages than actual nonperformance. In this case, a court is likely to find "willful acts" to refer to conduct by the parties that is tortious in nature (i.e., wrongful conduct in which the defendant willfully intends to inflict harm on the plaintiff at least in part through the means of breaching the contract between the parties) rather than merely refusing to continue to perform. Therefore, the court will likely not find a willful act here, and consequential damages will not be available under the contract. Thus, answer choice A is incorrect.

Under New York law, may a mother recover damages for emotional harm when medical malpractice causes a miscarriage or stillbirth?


A. No, unless the evidence demonstrates mother suffered a legally cognizable physical injury distinct from the fetusB. Yes, provided the evidence demonstrates a physical injury to the mother distinct from the incidences of childbirthC. No, because any violation of duty is to the fetus rather than to the motherD. Yes, because medical malpractice resulting in miscarriage or stillbirth is a violation of a duty of care to the expectant mother, entitling her to damages for emotional distress

Answer choice D is correct. New York courts allow a mother to recover for emotional injuries when medical malpractice caused a stillbirth or a miscarriage, even without a showing that she suffered an independent physical injury. In treating a pregnancy, medical professionals owe a duty of care to the developing fetus. They also owe a duty of reasonable care to the expectant mother, who is, after all, the patient. Thus, answer choice C is incorrect. Even in the absence of an independent injury, medical malpractice resulting in miscarriage or stillbirth should be construed as a violation of a duty of care to the expectant mother, entitling her to damages for the resulting emotional distress. Thus, answer choices A and B are incorrect.


Seller inherited a commercial building from his parents. Seller entered into a contract with Buyer to sell the inherited property for $100,000. Seller put down 10 percent of the purchase price and conducted a title examination. The result of the title examination revealed that the building was in violation of city zoning ordinances, thereby rendering the property unmarketable. Buyer has come to you to determine his remedies for Seller's inability to render marketable title. Which of the following is an unavailable remedy?


A. Recover the purchase money paid.B. Specific performance with an abatement of the purchase price to compensate for the defect in title.C. Expenses of the title examination.D. Nominal damages.

Answer choice B is the correct answer. In New York, if title is unmarketable and the seller is not at fault, then a buyer's remedies are limited to recovering the purchase money paid, expenses of the title examination, and nominal damages. Thus, answer choices A, C, and D are incorrect.


Defendant and Victim were drinking beer in the hallway of Defendant's six-story apartment building. At one point, a disagreement escalated into an argument. Victim raised a knife toward Defendant, and Defendant responded by pulling a gun from inside his jacket and shooting Victim dead. Defendant has asked you, as his attorney, to advise him as to whether he may raise justification as an affirmative defense.


A. Defendant can raise a justification defense because the shared hallway is part of Defendant's dwelling.B. Defendant cannot raise a justification defense because he was not actually inside his apartment when confronted by Victim.C. Defendant can raise the justification defense provided he establishes Victim initiated the argument.D. Defendant cannot raise the justification defense because a person is required to abandon or retreat from his home before causing the death of another.

Answer choice B is correct. Pursuant to N.Y. Penal Law § 35, New York recognizes a justification defense to homicide when a person is in his own home. Thus, answer choice D is incorrect. The justification defense makes no distinction over which party initiated a confrontation. Thus, answer choice C is incorrect. Nonetheless, the justification defense and the absence of a duty to retreat apply only to a person's dwelling, when the person exercises exclusive possession and control, and it does not extend to a shared hallway because Defendant is capable of going into his apartment and closing a door rather than resorting to violence. Thus, answer choice A is incorrect.

Testator created a testamentary trust naming Trustee to manage assets for the benefit of Testator's grandchildren, Beneficiaries. The trust expressly provided an exculpatory provision promising not to attempt to hold Trustee liable for any mismanagement of funds. After Testator died, Trustee in good faith made some high-risk investments on behalf of Beneficiaries. The investments failed, and Trustee lost 90% of the principal. Beneficiaries sought to file suit against Trustee for breach of his fiduciary duties. Trustee moved to dismiss.

How should a court rule?


A. Dismiss the action because the exculpatory clause precludes suit.B. Dismiss the action provided that Trustee acted in good faith.C. Deny the motion to dismiss because the exculpatory clause is invalid.D. Deny the motion upon a showing of negligence.

Answer choice C is correct. Courts tend to strictly interpret exculpatory clauses that relieve trustees from liability. Complete exculpatory clauses are void as contrary to public policy, and limited clauses are only permitted when there is no bad faith or unreasonableness. However, New York holds any exculpatory clause to be void with regard to testamentary trusts. The trust at issue here is a testamentary trust, thus the exculpatory clause cannot be invoked irrespective of whether it's complete or limited to instances when there is no bad faith. Thus, answer choices A, B, and D are incorrect.

After allegedly participating in an attack against three individuals at a subway station, then 15-year-old Alfred A and 14-year-old Barry B were charged with multiple counts of assault in varying degrees, the highest being two counts of assault in the first degree.

Over which, if either, of the juveniles may the criminal court exercise jurisdiction for the crimes charged?


A. Both Alfred A and Barry BB. Alfred A onlyC. Barry B onlyD. Neither

Answer choice A is correct. In New York, for certain crimes, including assault, 14-year-olds and 15-year-olds are subjected to prosecution in criminal court rather than to a proceeding in the family court. The New York legislature has divested family court of original jurisdiction by creating a class of "juvenile offenders" consisting of 14- and 15-year-olds (and, in a few instances, 13-year-olds) who are accused of committing one of the specifically named serious violent felonies. See N.Y. Penal Law § 10.00(18). As such, answer choice A is correct, and answer choices B, C, and D are incorrect.

Plaintiff entered into an agreement with Agent to transport materials to Defendant. Agent misrepresented to Defendant that he was authorized to collect payments on behalf of Plaintiff. Defendant contacted Plaintiff to confirm that Agent worked for Plaintiff, and Plaintiff's personnel confirmed that Agent was in "good standing" with Plaintiff. Agent kept monies received from Defendant, and Plaintiff filed a lawsuit against Defendant for nonpayment.

Will Plaintiff prevail in its suit for nonpayment against Defendant?


A. No, because Plaintiff was liable for the fraudulent acts of its agent.B. No, because Defendant was reasonable in concluding that collecting payments was a responsibility incidental to transporting materials.C. Yes, because collecting payments is incidental to the responsibility for transporting items and imputed implied authority onto Agent.D. Yes, because Agent had neither actual nor apparent authority to accept payments on Plaintiff's behalf.

Answer choice D is correct. There is no basis to conclude that the contract between Plaintiff and Agent endowed Agent with actual authority to accept payments on Plaintiff's behalf. The agreement authorized transport; nothing in the agreement authorized collection of payments. Nor is there support for a determination that Agent had apparent authority to collect payments on behalf of Plaintiff. Apparent authority is what the principal, through word or deed, causes a third party to reasonably believe about the agent's authority to act, and a principal is bound by a third party's reasonable reliance upon the appearance of authority. Here, confirmation that Agent is in good standing in no way suggests authority to collect payment. Thus, answer choice B is incorrect. Answer choice C is incorrect because nothing suggests that Plaintiff expressly or impliedly conferred authority on the Agent to collect money. Answer choice A is incorrect because an agent's fraud is not within the scope of any authority that can be imputed to the principal. An agent is always subject to liability to a third party harmed by the agent's tortious conduct, regardless of whether the principal is also liable.

Plaintiff, a New York resident, was involved in a motor vehicle accident with Defendant, a New Jersey resident, while driving through Connecticut. Plaintiff filed suit in New York Supreme Court against Defendant. After discovery was completed, the case was set for trial. Immediately prior to trial, Defendant filed a motion to dismiss based on lack of personal jurisdiction. But for the accident with Plaintiff in Connecticut, Defendant had had no contact with the state of New York. How should the court rule?


A. For Defendant based on lack of personal jurisdictionB. For Plaintiff, because a non-resident driver subjects himself to personal jurisdiction by committing a tortious act on a New York residentC. For Defendant because commission of a single tortious act on a New York resident is insufficient to establish personal jurisdictionD. For Plaintiff because Defendant waived the personal jurisdiction defense

Answer choice D is correct. A defendant must object to lack of personal jurisdiction in a pre-answer motion to dismiss or in the answer; otherwise, a defendant is deemed to have waived the objection with respect to that action. A defendant waives the personal jurisdiction defense by appearing in an action, or by conduct, express submission, or extended inaction. If a party makes voluntary appearances and contests the case at all stages, such conduct gives a court jurisdiction. Here, because Defendant had participated in all stages up through discovery, Defendant waived the objection to personal jurisdiction. Thus, answer choice A is incorrect.

In September, Plaintiff loaned $64,000 to Defendant. The promissory note bore an annual interest rate of 16%. In addition, Plaintiff charged reasonable expenses for processing the loan, which raised the actual rate of interest paid to 17%. One year later, Defendant defaulted, and Plaintiff commenced the present action for non-payment. How should the court rule?


A. For Plaintiff, because the contract is enforceable.B. For Defendant, because charging fees for a loan is a violation of the usury laws.C. For Plaintiff, because despite the contract being void for illegality, Plaintiff has a right to recover under quasi-contract for unjust enrichment.D. For Defendant because the loan is usurious and void.

Answer choice A is correct. New York law explicitly makes certain contracts usurious and unenforceable when the interest rate exceeds 16% per annum. However, a borrower may pay reasonable expenses attendant on a loan without rendering the loan usurious. Thus, the loan here is not usurious and answer choices B, C, and D are incorrect.

Defendant was on trial for kidnapping Victim. The entire incident lasted approximately 35 to 40 minutes before Victim escaped. Victim was traumatized by the event and had difficulty recollecting details relating to what Defendant looked like. She went under hypnosis to assist her in recollecting details of the kidnapping incident, including details related to Defendant's appearance. At trial, Victim testified against Defendant. Several inconsistencies arose between Victim's description of Defendant and what she had testified to under hypnosis. Specifically, at trial Victim testified that her assailant's blue shirt had the name "Mike's Gas Station" on it; when under hypnosis, she had stated that her assailant's shirt bore the name "Billie." Is the testimony from the hypnosis admissible evidence?


A. No.B. Yes, but for impeachment purposes only.C. Yes, but only as substantive evidence of the name that appeared on Defendant's shirt.D. Yes, for substantive and impeachment purposes.

Answer choice A is correct. In New York, substantive evidence testimony affected by hypnosis is nonprobative and inadmissible because hypnosis is not generally accepted as reliable in the scientific community. Hypnosis is not considered a reliable method of refreshing recollection because the hypnotic procedure is inherently suggestive. Thus, both answer choices C and D are incorrect. Evidence that is inadmissible as substantive evidence can be admissible on cross-examination for impeachment purposes. Thus, prior inconsistent statements, which might be inadmissible hearsay if introduced on direct examination, are admissible for impeachment purposes. A statement made under hypnosis may be the product of suggestion and not fairly the witness's own. Even if such a statement is inconsistent with a witness's trial testimony, because

Lawyer represented Client in a custody dispute. Client’s adversary, Former Spouse, was self–represented.

Which of the following is least accurate regarding Lawyer’s ethical obligations concerning communications with Former Spouse?


A. Lawyer need not advise Former Spouse to retain counsel.B. Lawyer may identify the legal issues for Former Spouse that could be usefully addressed by counsel.C. Lawyer may provide incontrovertible factual or legal information to Former Spouse.D. Lawyer may engage in any communication with Former Spouse provided such communication is neither deceptive nor misleading.

Answer choice D is correct. Attorneys acting on behalf of a client may have to deal directly with self-represented persons in a wide variety of transactional and litigation contexts. A lawyer engaging in any of these communications should remain mindful of the need to avoid misleading the self-represented party. A lawyer should be ready, when dealing with a self-represented person, to clarify when needed (i) that the lawyer does not and cannot represent the self-represented person, (ii) that the lawyer represents another party in the matter who may have interests adverse to the self-represented person, and (iii) that the lawyer cannot give the self-represented person any legal advice, other than to secure counsel or to consult an available court facility designed to assist self-represented persons. Although a lawyer may advise an unrepresented party to retain counsel, he is not required to. Thus, answer choice A is accurate. A lawyer may also identify the legal issues that could be usefully addressed by counsel and provide incontrovertible factual or legal information to an unrepresented party. Thus, answer choices B and C are accurate. Answer choice D is inaccurate because Lawyer’s communication with Former Spouse is significantly limited by the fact that he may not give her legal advice.

Husband and Wife divorced after 35 years of marriage. Husband was ordered to pay spousal maintenance for a period of 10 years. After the divorce was granted, Wife began living with another man with whom she acknowledged having sexual relations. Husband moved to terminate his obligation to continue making spousal payments on the grounds that Wife was cohabitating with another man, thereby relieving him of further obligation to pay spousal maintenance. How should the court rule?


A. For Husband because cohabitation terminates a maintenance obligation.B. For Husband because cohabitation constitutes holding oneself out as the spouse of a new domestic partner, thereby relieving the former spouse of any maintenance obligation.C. For Wife because cohabitation alone does not, in and of itself, constitute holding oneself out as the spouse of another.D. For Wife because only the death of either spouse or a legal marriage terminates a maintenance obligation.

Answer choice C is correct. Absent an agreement to the contrary, spousal maintenance is generally terminated automatically upon the death of either spouse or remarriage. Additionally, under New York law, a spousal maintenance obligation may be terminated when circumstances exist in which the spouse cohabitates with another and holds herself out as if she were married to the cohabitant. It is well established by case law that both elements of the statute must be established. Mere cohabitation is insufficient. Thus, answer choices A, B, and D are incorrect.

After jury deliberations in a felony criminal trial commenced, a juror became ill and was hospitalized. Defendant objected to a mistrial, and made a written request of the court to allow deliberations to continue with the remaining eleven jurors. The court granted the request. Subsequently, Defendant was convicted and appealed the decision, claiming that a verdict by fewer than twelve jurors violated his due-process rights. How should the appellate court rule?


A. Against Defendant, because his written consent to a deliberating jury of fewer than 12 was valid.B. For Defendant, because a jury of fewer than 12 hinders a fair trial.C. Against Defendant, because he was convicted.D. For Defendant, because a conviction by fewer than 12 jurors deprives a Defendant of due process of law.

Answer choice A is correct. Under New York law, a felony trial is required to have 12 jurors. However, when a juror becomes unavailable, a defendant may waive a 12 person jury and proceed with 11 and still have a fair trial. Thus answer choice B is incorrect. The U.S. Constitution only finds a jury of fewer than 6 a denial of due process, and thus D is incorrect. Answer choice C is incorrect because the outcome of an appeal in a criminal matter does not turn on whether a defendant was convicted.


Decedent died intestate survived by her husband, Father, and two adult children, Robin and Nancy. Robin filed a petition for letters of administration estimating the value of the estate at $900,000. Thereafter, Father, who is 85 years of age, filed a petition for letters of administration estimating the value of Decedent’s estate at $250,000. Nancy executed and filed a waiver of citation, consenting that letters of administration issue to Father.

Which party should be granted letters of administration?


A. Father, because he was Decedent’s husband.B. Father, because he received the consent of the other interested party, Nancy.C. Robin, because she was Decedent’s issue.D. Robin, because Father’s age renders him presumptively incompetent.

Answer choice A is correct. In this fact pattern, the only relevant classes of individuals deemed ineligible to serve as fiduciaries are those who are "want of understanding" or "otherwise unfit for the execution of the office." Age alone does not render a person ineligible as a matter of law and therefore answer choice D is incorrect. There is a statutorily mandated order of priority for granting letters of administration. Here, Decedent was survived by a spouse, Father, who is also entitled to share in the estate. Thus, he takes precedence over all others, including issue, and answer choice C is incorrect. Answer choice B is incorrect because an interested party's consent is not recognized as pertinent to establishing priority.